You are on page 1of 69

1.

Article 262 of the Constitution provides Additional Information:


for the adjudication of inter-state water
• The Inter-State Water Disputes Act
disputes by a separate tribunal. The need for
empowers the Central government to set
extrajudicial machinery to settle inter-state
up an ad hoc tribunal to adjudicate a
water disputes is because:
dispute between two or more states about
A. The courts being overburdened with an inter-state river or river valley's
litigations and adjudicating slowly are waters.
incapable of adjudicating sensitive
• The decision of the tribunal would be final
water disputes
and binding on the parties to the dispute.
B. Water resources are not private
• Neither the Supreme Court nor any other
property, and so the rule of law
court is to have jurisdiction regarding any
applied by ordinary courts is not
water dispute that may be referred to such
appropriate to deal with its
a tribunal under this Act.
distribution
C. Division and distribution of any
natural resources is out of judicial 2. The Indian Constitution has provisions
scrutiny in India related to which of the following?

D. The courts cannot employ technical 1. Adjudication of inter-state water


committees to ascertain the disputes
distribution of water resource 2. Coordination between Centre and
Answer: B State through inter-state councils

Explanation: 3. Freedom of inter-state trade,


commerce and intercourse
Option B is correct:
Select the correct answer using the codes
The need for extra judicial machinery to settle
below:
inter-state water disputes is as follows:
A. 1 and 2 only
• The Supreme Court would indeed have
jurisdiction to decide any dispute between B. 3 only
states in connection with water supplies if C. 2 and 3 only
legal rights or interests are concerned, but
D. 1, 2 and 3
the experience of most countries has
shown that rules of law based upon the Answer: D
analogy of private proprietary interests in
water do not afford a satisfactory basis
for settling disputes between the states
where the interests of the public at large in
the proper use of water supplies are
involved.
Explanation: • Option a is incorrect: The Zonal Councils
• Statement 1 is correct: Article 262 of the are the statutory (and not the
Constitution provides for the adjudication constitutional) bodies. They are
of interstate water disputes. Established by an Act of the Parliament,
that is, States Reorganisation Act of 1956.
o It makes the provision that
Parliament may by law provide for Additional Information:
the adjudication of any dispute or • The act divided the country into five zones
complaint concerning the use, Northern, Central, Eastern, Western and
distribution and control of waters of Southern and provided a zonal council for
any inter-state river and river valley. each zone.
• Statement 2 is correct: Article 263 • In addition to the above-mentioned Zonal
contemplates establishing an Inter-State Councils, a North-Eastern Council was
Council to affect coordination between the created by a separate Act of Parliament,
states and between Centre and states. the North-Eastern Council Act of 1971.
• Statement 3 is correct: Articles 301 to 307 • These councils are advisory bodies that
in Part XIII of the Constitution deal with make recommendations with regard to
the trade, commerce, and intercourse any matter of common interest in the field
within India's territory. Article 301 of economic and social planning between
declares that trade, commerce and the Centre and States border disputes,
intercourse throughout the territory of linguistic minorities, inter-State transport
India shall be free. or matters connected with the
reorganization of States.

3. Which of the following statements with


reference to the Zonal Councils in India is 4. Consider the following statements:
incorrect?
1. The President can delegate executive
A. Provision for Zonal Councils has been functions of the Centre to the State,
provided under Part XIII of the without the consent of the State.
constitution
2. The Governor of State can delegate
B. There are five zonal councils formed executive functions of State to the
under States Reorganisation Act 1956 Centre, with the consent of the Central
C. The North Eastern Zonal Council was Government.
not created under States Which of the statements given above is/are
Reorganisation Act 1956 correct?
D. Zonal councils are advisory bodies that A. 1 only
make recommendations with regard to
B. 2 only
any matter of common interest
C. Both 1 and 2
Answer: A
D. Neither 1 nor 2
Answer: B 5. The Chairman of the interstate council is:
Explanation: A. Prime Minister
• Statement 1 is incorrect: The President can B. Chief Ministers of different states on
delegate executive functions of the Centre rotation
to the State, with the consent of the State. C. President
• Statement 2 is correct: The Governor of a D. Chairman of the Planning Commission
state may (with the consent of the Central
Answer: A
govt.) entrust to that govt. any of the
executive functions of the state. Explanation:
Additional Information: Option a is correct:

• Mutual Delegation of Functions • Article 263 provides establishment of


Inter-State Council
o The distribution of executive
powers between Centre and State is • Sarkaria Commission on centre-state
rigid and many times lead to relations recommended establishment of a
conflicts, hence the Constitution permanent interstate council.
provides for inter-government
• Therefore, in 1990 Inter-State Council was
delegation of executive functions to established with Prime Minister as its
mitigate rigidity & avoid a situation chairman
of deadlock.
o Accordingly, the President may
6. The North Eastern Council is headed by
(with the consent of the state govt.)
which of the following?
entrust to that govt. any of the
executive functions of the Centre. A. President of India

o Conversely, Constitution also B. Prime Minister


makes a provision for the C. Union Home Minister
entrustment of the executive
D. Minister of State, Ministry for
functions of the Centre to a state
Development of North Eastern Region
(without the consent of that state)
(DoNER)
here, the delegation is by Parliament
(not by President) Answer: C

o Mutual delegation of functions Explanation:


between the Centre and State can
take place either under an
agreement or by legislation. (Centre
can use both the methods but a state
can use only the agreement method)
Recent amendments: C. Part XIII
• The Union Cabinet, in June 2018, D. Part X
approved the proposal of the Ministry of Answer: C
Development of North Eastern Region
(DoNER) for the nomination of the union Explanation:
home minister as ex-officio Chairman of • Option c is correct: Part XIII (Articles 301-
North Eastern Council (NEC). 307) deals with freedom of trade,
commerce and intercourse. The articles in
• The Cabinet has also approved that the
this part provide for power of Parliament
Minister of State (Independent Charge),
to impose restrictions on trade, commerce
Ministry of DoNER would serve as Vice
and intercourse and restrictions on the
Chairman of the Council.
legislative powers of the Union and of the
Option c is correct: Under the new arrangement, States with regard to trade and commerce.
the Home Minister shall be the Chairman and
Minister of DoNER as Vice Chairman, NEC and all
the Governors and Chief Ministers of North 8. Consider the following statements with
Eastern States will be Members. reference to parliamentary form of democracy:
Additional Information: 1. President is the head of the state and
the real executive.
• NEC was established under the North
Eastern Council Act, 1971 as an apex level 2. Prime minister is the head of the
body for securing balanced and government and the nominal
coordinated development and facilitating executive.
coordination with the States. 3. Collective responsibility is the
• Subsequent to the Amendment of 2002, fundamental principle of
NEC has been mandated to function as a parliamentary form of democracy.
regional planning body for the North Which of the statements given above is/are
Eastern Area and while formulating a incorrect?
regional plan for this area, shall give
A. 1 and 2 only
priority to the schemes and projects
benefiting two or more states provided B. 2 and 3 only
that in the case of Sikkim, the Council shall C. 1 and 3 only
formulate specific projects and schemes
D. 1, 2 and 3
for that State.
Answer: A
Explanation:
7. Which part of the Indian constitution
deals with Trade, Commerce and Inter Statement 1 and 2 are incorrect:
Course within the territories of India?
• The President is the nominal executive (de
A. Part XI jure executive or titular executive) while
B. Part XII the Prime Minister is the real executive (de
facto executive). C. 3 only
• Thus, the President is head of the State, D. 4 only
while the Prime Minister is head of the Answer: B
government.
Explanation:
• Hence, statement 1 and statement 2 are
Statement 2 and 3 are correct:
incorrect.
In spite of the above merits, the parliamentary
Statement 3 is correct:
system suffers from the following demerits:
• Article 74 provides for a council of
• Unstable Government: The parliamentary
ministers headed by the Prime Minister to
system does not provide a stable
aid and advise the President in the
government. There is no guarantee that a
exercise of his functions.
government can survive its tenure.
• The advice so tendered is binding on the
o The ministers depend on the mercy
President.
of the majority legislators for their
• The ministers are collectively responsible continuity and survival in office.
to the Parliament in general and to the Lok The Government headed by Morarji
Sabha in particular (Article 75). Desai, Charan Singh, V.P. Singh,
• They act as a team, and swim and sink Chandra Sekhar, Deva Gowda and
together. I.K. Gujral are some such examples.

• The principle of collective responsibility • No Continuity of Policies: The


implies that the Lok Sabha can remove the parliamentary system is not conducive for
ministry (i.e., council of ministers headed the formulation and implementation of
by the prime minister) from office by long-term policies. This is due to the
passing a vote of no confidence. uncertainty of the tenure of the
government. For example, the Janata
• Hence, statement 3 is correct. Government headed by Morarji Desai in
Source: Laxmikant (Parliamentary System). 1977 reversed a large number of policies of
the previous Congress Government.

9. Which of the following are some of the • Dictatorship of the Cabinet: When the
drawbacks of the Parliamentary System? ruling party enjoys absolute majority in
the Parliament, the cabinet becomes
1. Wide Representation
autocratic and exercises nearly unlimited
2. Dictatorship of the Cabinet powers.
3. Government by amateurs • Against Separation of Powers: In the
4. Responsible Government parliamentary system, the legislature and
the executive are together and
How many of the above statements are correct?
inseparable. The cabinet acts as the leader
A. 1 only of the legislature as well as the executive.
B. 2 only
• Government by Amateurs: The of individuals (i.e., ministers who are
parliamentary system is not conducive to representatives of the people). Hence, it is
administrative efficiency as the ministers possible to provide representation to all
are not experts in their fields. The Prime sections and regions in the government.
Minister has a limited choice in the Source: Laxmikant (Parliamentary System).
selection of ministers; his choice is
restricted to the members of Parliament
alone and does not extend to external 10. Consider the following with reference to the
talent. effect of the National emergency on fundamental
rights:
Hence, option B is correct.
1. When a proclamation of National
The parliamentary system of government has the
following merits: emergency is made 6 rights under
article 19 are automatically suspended.
• Harmony Between Legislature and
2. In case National emergency is
Executive: The executive is a part of the
proclaimed on the ground of external
legislature and both are interdependent at
aggression, fundamental rights under
work. As a result, there is less scope for
article 19 are not suspended.
disputes and conflicts between the two
organs. 3. The President in such times is also
empowered to suspend the right to
• Responsible Government: By its very
move any court for enforcement of any
nature, the parliamentary system
fundamental rights except article 20
establishes a responsible government. The
and article 21.
ministers are responsible to the Parliament
for all their acts of omission and Which of the statements given above is/are
commission. correct?

• Prevents Despotism: Under this system, A. 1 and 2 only


the executive authority is vested in a B. 2 and 3 only
group of individuals (council of ministers)
C. 1 and 3 only
and not in a single person. This dispersal
of authority checks the dictatorial D. 1, 2 and 3
tendencies of the executive. Answer: C
• Ready Alternative Government: In case Explanation:
the ruling party loses its majority, the
• Statement 1 is correct: According to
Head of the State can invite the opposition
Article 358, when a proclamation of
party to form the government. This means
national emergency is made, the six
an alternative government can be formed
Fundamental Rights under Article 19
without fresh elections.
are automatically suspended. Hence,
• Wide Representation: In a parliamentary statement 1 is correct.
system, the executive consists of a group
• Statement 2 is incorrect: The 44th
Amendment Act of 1978 restricted the in the assembly and recommends
scope of Article 358. The six imposition of President’s Rule without
Fundamental Rights under Article 19 allowing the ministry to prove its
can be suspended only when the majority on the floor of the Assembly.
National Emergency is declared on the Answer: C
ground of war or external aggression
and not on the ground of armed Explanation:
rebellion. Hence, statement 2 is Option C is correct:
incorrect. Based on the report of the Sarkaria Commission on
• Statement 3 is correct: Article 359 Centre-state Relations (1988), the Supreme Court in
authorizes the president to suspend Bommai case (1994) enlisted the situations where
the right to move any court for the the exercise of power under Article 356 could be
enforcement of Fundamental Rights proper or improper. Imposition of President’s
during a National Emergency. The Rule in a state would be proper in the following
44th Amendment Act of 1978 restricted situations:
the scope of Article 359 in two ways. • Where after general elections to the
Firstly, the President cannot suspend assembly, no party secures a majority,
the right to move the Court for the that is, ‘Hung Assembly’.
enforcement of fundamental rights
• Where the party having a majority in
guaranteed by Articles 20 to 21. Hence,
the assembly declines to form a
statement 3 is correct.
ministry and the governor cannot find
Source: Laxmikant(Emergency Provisions) a coalition ministry commanding a
majority in the assembly.
11. Which of the following is a proper condition • Where a ministry resigns after its
for use of the President's Rule as per the Bommai defeat in the assembly and no other
case judgment of the Supreme Court? party is willing or able to form a
A. Where the ruling party enjoying ministry commanding a majority in the
majority support in the assembly has assembly.
suffered a massive defeat in the • Where the constitutional direction of
general elections to the Lok Sabha. the Central government is
B. Maladministration in the state or disregarded by the state government.
allegations of corruption against the Hence, option C is correct.
ministry or stringent financial • Internal subversion where, for
exigencies of the state. example, a government is deliberately
C. Where a constitutional direction of the acting against the Constitution and the
Central government is disregarded by law or is fomenting a violent revolt.
the state government • Physical breakdown where the
D. Where the governor makes his own government wilfully refuses to
assessment of the support of a ministry discharge its constitutional obligations
endangering the security of the state. Source: Laxmikant (Emergency
Option A, B and D are incorrect: Provisions).

• The imposition of President’s Rule in a


state would be improper under the 12. Consider the following with reference to
following situations: Financial Emergency:
1. A proclamation declaring financial
• Where a ministry resigns or is
emergency must be approved by both
dismissed on losing majority support
in the assembly and the governor the Houses of Parliament within two
recommends imposition of President’s months from the date of its issue.
Rule without probing the possibility of 2. Once approved by both the Houses of
forming an alternative ministry. Parliament, the Financial Emergency
continues indefinitely till it is revoked.
• Where the governor makes his own
assessment of the support of a ministry 3. When a financial emergency is
in the assembly and recommends proclaimed, the President may issue
imposition of President’s Rule without directions for the reduction of salaries
allowing the ministry to prove its and allowances of the judges of the
majority on the floor of the Assembly. Supreme Court and the high court.

• Where the ruling party enjoying Which of the statements given above is/are
majority support in the assembly has correct?
suffered a massive defeat in the A. 1 and 2 only
general elections to the Lok Sabha such B. 2 and 3 only
as in 1977 and 1980.
C. 1 and 3 only
• Internal disturbances not amounting to
D. 1, 2 and 3
internal subversion or physical
breakdown. Answer: D

• Maladministration in the state or Explanation:


allegations of corruption against the • Article 360 empowers the president to
minis-tryor stringent financial proclaim a Financial Emergency if he is
exigencies of the state. satisfied that a situation has arisen due to
which the financial stability or credit of
• Where the state government is not India or any part of its territory is
given prior warning to rectify itself threatened.
except in case of extreme urgency • Statement 1 is correct: A proclamation
leading to disastrous consequences. declaring financial emergency must be
approved by both the Houses of
• Where the power is used to sort out
Parliament within two months from the
intraparty problems of the ruling date of its issue. Hence, statement 1 is
party, or for a purpose extraneous or correct.
irrelevant to the one for which it has
• Statement 2 is correct:
been conferred by the Constitution.
o Once approved by both the Houses allowances of all or any class of
of Parliament, the Financial persons serving in the state;
Emergency continues indefinitely o the reservation of all money bills or
till it is revoked. Hence, statement 2 other financial bills for the
is correct. consideration of the President after
• Statement 3 is correct: The President may they are passed by the legislature of
issue directions for the reduction of the state.
salaries and allowances of Source: Laxmikant (Emergency Provisions).
o all or any class of persons serving
the Union;
13. Consider the following statements with
o the judges of the Supreme Court and reference to the 73rd Constitutional
the high court. Hence, statement 3 is Amendment Act.
correct.
1. The act gives a constitutional status to
Additional Information: the Panchayati Raj Institutions.
This implies two things: 2. The act has given a practical shape to
• there is no maximum period prescribed Article 40 of the Constitution.
for its operation; and 3. The act added the 11th schedule to the
• repeated parliamentary approval is not constitution.
required for its continuation. Which of the statements given above is/are
A proclamation of Financial Emergency may be correct?
revoked by the president at any time by a A. 1 and 2 only
subsequent proclamation. Such a proclamation
B. 2 and 3 only
does not require parliamentary approval.
C. 1 and 3 only
The consequences of the proclamation of a
Financial Emergency are as follows: D. 1, 2 and 3

• The executive authority of the Centre Answer: D


extends to the giving of Explanation:
o directions to any state to observe 73RD AMENDMENT ACT OF 1992
such canons of financial propriety as Statement 1 is correct: This act has added a
may be specified in the directions; new Part-IX to the Constitution of India. This
o such other directions to any state as part is entitled as ‘The Panchayats’ and
the President may deem necessary consists of provisions from Articles 243 to 243
and adequate for the purpose. O.

• Any such direction may include a Statement 3 is correct: In addition, the act has
provision requiring also added a new Eleventh Schedule to the
Constitution. This schedule contains 29
o the reduction of salaries and
functional items of the panchayats. It deals
with Article 243-G. Hence, statement 3 is D. It is a body of representatives elected
correct. by people who are residents of the
village.
Statement 2 is correct: The act has given a
practical shape to Article 40 of the Answer: A
Constitution which says that, “The State shall Explanation:
take steps to organize village panchayats and
• The 73rd CAA is a significant
endow them with such powers and authority
landmark in the evolution of grassroot
as may be necessary to enable them to
democratic institutions in the country.
function as units of self-government.” This
article forms a part of the Directive Principles • It transfers representative democracy
of State Policy. Hence, statement 2 is correct. into participatory democracy. It is a
Additional info: revolutionary concept to build
democracy at the grassroot level in the
The act gives a constitutional status to the country.
panchayati raj institutions. It has brought
them under the purview of the justiciable part • The act provides for a Gram Sabha as
of the Constitution. In other words, the state the foundation of the panchayati raj
governments are under constitutional system.
obligation to adopt the new panchayati raj • It is a body consisting of persons
system in accordance with the provisions of registered in the electoral rolls of a
the act. Consequently, neither the formation village within the area of Panchayat at
of panchayats nor the holding of elections at the village level.
regular intervals depend on the will of the
• Thus, it is a village assembly consisting
state government any more. Hence, statement
of all the registered voters in the area
1 is correct.
of a panchayat. It may exercise such
Source: Laxmi Kant (73rd CAA). powers and perform such functions at
the village level as the legislature of a
state determines. Hence, option A is
14. Which of the following is correct
correct.
regarding 'Gram Sabha'?
Source: Laxmi kant (73rd CAA).
A. It is a body consisting of people
registered in electoral rolls of the
village. 15. Consider the following with reference to
B. It is a body consisting of people who Finances of Rural Local Bodies:
are residents of the village. 1. Governor of the state is obliged to set
C. It is a body of representatives elected up a state finance commission every 5
by people registered in electoral rolls years to review the financial position of
of the village. panchayats.
2. The central finance commission is not
allowed to look into the distribution of
finances between State Government The principles that should govern:
and local governments. • The distribution between the state and
Which of the statements given above is/are the panchayats of the net proceeds of
correct? the taxes, duties, tolls and fees levied
A. 1 only by the state and allocation of shares

B. 2 only • amongst the panchay-ats at all levels.

C. Both 1 and 2 • The determination of taxes, duties,


tolls and fees that may be assigned to
D. Neither 1 nor 2 the panchayats.
Answer: A • The grants-in-aid to the panchayats
Explanation: from the consolidated fund of the
Statement 1 is correct: The governor of a state state.
shall, after every five years, constitute a The measures needed to improve the financial
finance commission to review the financial position of the panchayats.
position of the panchayats. Hence, statement
Any other matter referred to by the governor
1 is correct.
in the interests of sound finance of the
Statement 2 is incorrect: The Central Finance panchayats. The state legislature may provide
Commission shall also suggest the measures for the composition of the commission, the
needed to augment the consolidated fund of a required qualifications of its members and the
state to supplement the resources of the manner of their selection. The governor shall
panchayats in the states (on the basis of the place the recommendations of the
recommendations made by the finance commission along with the action taken
commission of the state). Hence, statement 2 report before the state legislature.
is incorrect.
Source: Laxmikant (73rd CAA)
Additional info:
According to 73rd constitutional amendment
16. Which of the following factors are
act, the state legislature may (a) authorise a
considered by the governor to decide the
panchayat to levy, collect and appropriate type of urban local government for a
taxes, duties, tolls and fees; (b) assign to a particular area?
panchayat taxes, duties, tolls and fees levied
1. Density of the population
and collected by the state government; (c)
provide for making grants-in-aid to the 2. Economic importance
panchayats from the consolidated fund of the 3. Revenue generated for local
state; and (d) provide for constitution of government
funds for crediting all money of the
panchayats. Which of the statements given above is/are
correct?
It shall make the following
recommendations to the Governor: A. 1 and 2 only
B. 2 and 3 only 17. Which of the following functional items
are placed under the purview of
C. 1 and 3 only
municipalities under 12th schedule?
D. 1, 2 and 3
1. Planning for economic and social
Answer: D development.
Explanation: 2. Safeguarding the interests of weaker
The 74th constitutional amendment act sections of society
provides for the constitution of the following 3. Cattle ponds
three types of municipalities in every state.
4. Libraries
• A nagar panchayat (by whatever name
How many of the above items are correct?
called) for a transitional area.
A. 1 only
• A municipal council for a smaller
urban area. B. 2 only

• A municipal corporation for a larger C. 3 only


urban area. D. 4 only
But there is one exception. If there is an urban Answer: C
area where municipal services are being
Explanation:
provided by an industrial establishment, then
the governor may specify that area to be an Eleventh Schedule
industrial township. In such a case, a It contains the following 29 functional items
municipality may not be constituted. placed within the purview of panchayats:
The governor has to specify a transitional 1. Agriculture, including agricultural
area, a smaller urban area or a larger urban extension
area, keeping in view the following factors:
2. Land improvement, implementation
• Population of the area. of land reforms, land consolidation
• Density of the population therein. and soil conservation

• Revenue generated for local 3. Minor irrigation, water management


administration. and watershed development

• Percentage of employment in non- 4. Animal husbandry, dairying and


agricultural activities. poultry

• Economic importance. 5. Fisheries

• Such other factors as he may deem fit. 6. Social forestry and farm forestry

Hence, option D is correct. 7. Minor forest produce

Source: Laxmikant (74rd CAA). 8. Small-scale industries, including food


processing industries
9. Khadi, village and cottage industries
10. Rural housing A. Article 112
11. Drinking water B. Article 111
12. Fuel and fodder C. Article 110
13. Roads, culverts, bridges, ferries, D. None of the above
waterways and other means of Answer: D
communication
Explanation:
14. Rural electrification, including
• The term "budget" is not explicitly
distribution of electricity
mentioned in the Indian Constitution.
15. Non-conventional energy sources The term used in the Constitution is
16. Poverty alleviation programme "Annual Financial Statement," which
refers to the budget.
17. Education, including primary and
secondary schools Additional info:

18. Technical training and vocational • The Annual Financial Statement, also
education known as the Union Budget, is a
critical document that outlines the
19. Adult and non-formal education
government's plans for revenue
20. Libraries (Falls under 11th schedule) collection and expenditure for the
21. Cultural activities upcoming financial year.

22. Markets and fairs • It is an important tool for the


government to plan its spending and
23. Health and sanitation including
to allocate resources for various
hospitals, primary health centres and
development programs and initiatives.
dispensaries
• In India, the Annual Financial
24. Family welfare
Statement is presented by the Union
25. Women and child development Finance Minister in the Parliament on
26. Social welfare, including welfare of the the last working day of February each
handicapped and mentally retarded year.

27. Welfare of the weaker sections, and in • The Statement provides a


particular, of the scheduled castes and comprehensive picture of the
the scheduled tribes government's estimated revenue and
expenditure for the next financial year,
28. Public distribution system
starting from April 1 and ending on
29. Maintenance of community assets. March 31 of the following year.
• The Annual Financial Statement is a
18. Which of the following articles find a detailed document that provides
mention of “budget” in the Constitution of information on the government's
India? revenue sources, such as taxes, duties,
and fees, and its estimated are recorded. The Public Account is
expenditure, including spending on maintained under the provisions of the
defense, education, healthcare, and Constitution of India and is governed
other social and economic programs. by the rules and regulations specified
in the Public Account (India) Rules,
• It also provides information on the
1950.
government's plans for borrowing,
both domestically and internationally, • The Public Account consists of two
and its plans for investment in sub-accounts: the Consolidated Fund
infrastructure development and other of India and the Contingency Fund of
key sectors. India.
• The Union Budget is an important • The Consolidated Fund of India is the
document that guides the main account where all government
government's financial policies and revenues, including taxes, duties, and
plans and provides a roadmap for the fees, are credited. The Contingency
country's economic development. Fund of India is a reserve fund created
to meet unforeseen expenditures and
emergencies.
19. Which article of the Indian Constitution
provides for the “Public Account of India”? • All money credited to the Public
Account is kept in the custody of the
A. Article 265
President of India and can be
B. Article 266 withdrawn only with the
C. Article 267 authorization of the Parliament.

D. Article 268 • The money in the Public Account


cannot be used for day-to-day
Answer: B operations of the government without
Explanation: the prior approval of the Parliament.
• The Public Account of India plays a
crucial role in maintaining the financial
stability of the government and
ensuring accountability and
transparency in the use of public
funds.
• The government's management of the
Public Account is subject to regular
audits by the Comptroller and Auditor
• The Public Account of India refers to a General of India (CAG), who reports to
set of accounts maintained by the the Parliament on the utilization of
government of India, where all non- funds from the Public Account.
budgetary receipts and expenditures
20. Which of the following statements is 2. Article 109 of the Indian Constitution
correct regarding the Joint sitting of Indian provides for a joint sitting of both
Parliament? Houses of Parliament.
A. The President can summon a joint Which of the above statements is/are
sitting only in case the bill has not correct?
elapsed because of the Lok Sabha’s
A. 1 only
dissolution.
B. 2 only
B. Article 107 of the Indian Constitution
provides for a joint sitting of both C. Both 1 & 2
Houses of Parliament. D. Neither 1 nor 2
C. The President can summon a joint Answer: A
sitting in case of a Constitutional
Statement 1 is correct: The quorum to
Amendment Bill.
constitute a joint sitting is 1/10th of the total
D. The first joint sitting was conducted to membership of Parliament.
resolve the deadlock arising due to the
Statement 2 is incorrect: Article 108 of the
Banking Service Commission Act
Indian Constitution provides for a joint sitting
(Repeal), 1978.
of both Houses of Parliament.
Answer: A
Additional info:
Option B is incorrect: Article 108 of the
• A Joint Sitting in Indian Parliament
Indian Constitution provides for a joint sitting
refers to a rare and extraordinary
of both Houses of Parliament.
legislative procedure in which both
Option C is incorrect: The President can Houses of Parliament, the Lok Sabha
summon a joint sitting only in case the bill has (House of the People) and the Rajya
not elapsed because of the Lok Sabha’s Sabha (Council of States), sit together
dissolution. The President cannot summon a to resolve a deadlock on the passage of
joint sitting in case of a Constitutional a bill.
Amendment Bill and a Money Bill.
• Joint Sittings are authorized by Article
Option D is incorrect: The first joint sitting 108 of the Constitution of India and can
was conducted to resolve the deadlock that be called by the President of India on
arose due to the Dowry Prohibition Bill 1961. the recommendation of the Prime
Minister.

21. Consider the following statements • Joint Sittings are called when the Lok
regarding joint sitting of Indian Parliament. Sabha and the Rajya Sabha are unable
to agree on the passage of a bill, and
1. The quorum to constitute a joint sitting
the bill is returned by the Rajya Sabha
is 1/10th of the total membership of
to the Lok Sabha with
Parliament.
recommendations for changes.
• In a Joint Sitting, the Lok Sabha and the Which of the above statements is/are
Rajya Sabha sit together as a single correct?
legislative body, with Members of A. 1 only
Parliament (MPs) from both Houses
B. 2 only
participating in the proceedings.
C. 3 only
• The bill is discussed and debated by
the MPs, and a vote is taken on its D. None of the above
passage. In a Joint Sitting, a bill is Answer: A
considered to have been passed if it
receives the support of a majority of Statement 1 is correct: The 44th Amendment
the total membership of both Houses Act allowed the media to publish the true
of Parliament. reports of the parliamentary proceedings
except the same related to the house’s secret
• Joint Sittings are a rare occurrence in sitting.
Indian parliamentary history, and
have only been held three times since Statement 2 is incorrect: No person (either a
the adoption of the Constitution in member or outsider) can be arrested, and no
1950. Joint Sittings are considered to be legal process (civil or criminal) can be served
a measure of last resort, and are used within the precincts of the House without the
only when the Lok Sabha and the Rajya permission of the presiding officer
Sabha are unable to resolve their Statement 3 is incorrect: Members of
differences through normal legislative Parliament are exempted from jury service in
procedures. civil cases.

22. Consider the following statements 23. Consider the following statements
regarding Parliamentary privileges. regarding the Sansad Nidhi Yojana.
1. The 44th Amendment Act allowed the 1. The scheme is administered by the
media to publish the true reports of the Ministry of Rural Development.
parliamentary proceedings except the
2. The district authority must inspect at
same related to the house’s secret
least 10% of all works under
sitting.
implementation every year.
2. No person (either a member or
3. The funds under Yojana are sanctioned
outsider) can be arrested, and no legal
on an yearly basis and the unused
process (except other than criminal)
funds cannot be carried forward to the
can be served within the precincts of
next year.
the House without the permission of
the presiding officer. Which of the above statements is/are
correct?
3. Members of Parliament are not
A. 1 only
exempted from jury service in civil
cases. B. 2 only
C. 3 only • Communication
D. All of the above State List Subjects:
Answer: B • Some of the important subjects are:
Explanation: • Public order
Statement 1 is incorrect: The scheme was • Police
administered by the Ministry of Rural
• Public health and sanitation
Development, but now it is administered by
the Ministry of Statistics and Programme • Hospitals and dispensaries
Implementation. • Betting and gambling
Statement 2 is correct: The district authority
must inspect at least 10% of all works under
25. Which of the following committees made a
implementation every year.
pitch for the establishment of a permanent Inter-
Statement 3 is incorrect: Funds are non- State Council?
lapsable in nature i.e. in case of non-release of
A. Arun Goel Committee
funds in a particular year it is carried forward
to the next year. B. Rajiv Mehrishi
C. Sarkaria Commission

24. Which of the following is not a State List D. Tarapore Commission


Subject? Answer: C
A. Public order Explanation:
B. Police Option c is correct:
C. Defence • The Sarkaria Commission on Centre-
D. Hospitals and dispensaries State Relations (1983–88) made a pitch
for the establishment of a permanent
Answer: C
Inter-State Council (under Art. 263) of
Explanation: the Constitution.
Union List Subjects: • Commission recommended Inter-State
Some of the important subjects are: Council must be called as the
Intergovernmental Council in order to
• Defence
differentiate from other bodies
• Army established under the same Art. 263.
• International Relations • It recommended that the Council
• Ports should be charged with the duties laid
down in 263 (B) and (C).
• Railways
• Highways
26. The President acquires which of the following Answer: C
extraordinary powers when the President’s rule is Explanation:
imposed in a state?
• Statement 1 is correct: Cadre
A. He can take up the functions of the
controlling authority for deputation of
state government and powers vested
Indian Police Service and Indian Forest
in the governor or any other executive
Service Officers (IFoS) is the Ministry
authority in the state.
of Home Affairs (MHA) and the
B. He can declare that the powers of the Environment Ministry respectively.
state legislature are to be exercised by
• Statement 2 is correct: In the country,
the parliament.
there are twenty-four cadres, as well as
C. He can take all other necessary steps three joint cadres: Assam-Meghalaya,
including the suspension of the Manipur-Tripura, and Arunachal
constitutional provisions relating to Pradesh-Goa-Mizoram-Union
anybody or authority in the state. Territories (AGMUT).
D. All of the above • Statement 4 is incorrect: Ranks of the
Answer: D Indian Administrative Service

Explanation:
• Option A, B and C all are correct. 28. Consider the following statements with
Hence Option d is the correct answer. reference to the Territory of India’ and ‘Union of
India’:
1. Parliament may by law admit into the
27. Consider the following statement with
Union, or establish, new States on such
reference to All India Services:
terms and conditions as it thinks fit.
1. Cadre controlling authority for
2. For creation or destruction of a state
deputation of Indian Police Service
the permission from the concerned
and Indian Forest Service Officers
state is mandatory under Indian
(IFoS) is the Ministry of Home Affairs
Constitution.
(MHA) and the Environment Ministry
respectively. Which of the above statements is/are correct?

2. In the country, there are twenty-four A. 1 only


cadres, as well as three joint cadres B. 2 only
Which of the above statements is/are correct? C. Both 1 and 2
A. 1 only D. Neither 1 nor 2
B. 2 only Answer: A
C. Both 1 and 2 Explanation:
D. Neither 1 nor 2 • Part I of the Indian Constitution is
titled The Union and its Territory.
• It includes articles from 1- 4. Part I is a B. 3 only
compilation of laws pertaining to the C. 2 only
constitution of India as a country and
the union of states that it is made of. D. 1, 2 and 3
Answer: D
• This part of the constitution contains
the law in the establishment, renaming, Explanation:
merging or altering the borders of the Some bills can be introduced in the Parliament only
states. Articles under Part I were on the recommendation of the President so as to
invoked when West Bengal was protect the financial interests of the states:
renamed, and for formation of
relatively new states such as • Statement 1 is correct: A bill which
Jharkhand, Chhattisgarh or Telangana. imposes or varies any tax or duty in
which states are interested
• Statement 1 is correct: Parliament may
by law admit into the Union, or • Statement 2 is correct: A bill which
establish, new States on such terms and affects the principles on which moneys
conditions as it thinks fit. are or may be distributable to states;
and
• Statement 2 is incorrect: For creation
or destruction of a state the permission • Statement 3 is correct: A which
from the concerned state is not imposes any surcharge on any
mandatory under Indian Constitution. specified tax or duty for the purpose of
But the bill has to be referred to the the centre
concerned state legislature for Other type:
expressing its views.
• A bill which varies the meaning of the
term ‘agricultural income’ as defined
29. Which of the following bills can be for the purposes of the enactments
introduced in the Parliament only on the relating to the income tax
recommendation of the President?
1. A bill which imposes or varies any tax 30. Consider the following statements with
or duty in which states are interested reference to GST Council:
2. A bill which affects the principles on 1. Each state can nominate a minister in-
which moneys are or may be charge of finance or taxation or any
distributable to states other minister as a member.
3. A which imposes any surcharge on any 2. In the GST council the vote of the
specified tax or duty for the purpose of central government shall have a
the centre weightage of one-third of the total
Select the correct answer using the codes given votes cast in the meeting.
below: Which of the above given statements is/are
A. 1 and 2 only correct?
A. 1 only 31. Which of the following articles in Part 4 of the
constitution provide for “equal justice and free
B. 2 only
legal aid to the poor”?
C. Both 1 and 2
A. Article 38
D. Neither 1 nor 2
B. Article 39A
Answer: C
C. Article 43
Explanation:
D. Article 43A
• Statement 1 is correct: Each state can
Answer: B
nominate a minister in-charge of
finance or taxation or any other Explanation:
minister as a member. • Option A is incorrect: Article 38 states
• Statement 2 is correct: In the GST that the State shall strive to promote
council the vote of the central the welfare of the people by securing
government shall have a weightage of and protecting as effectively as it may
one-third of the total votes cast in the a social order in which justice, social,
meeting. economic and political, shall inform all
the institutions of the national life.
Additional Information:
• Option B is correct: Article 39A- The
The decision is taken in accordance with the
State shall secure that the operation of
following principles:
the legal system promotes justice, on a
• The vote of the central government basis of equal opportunity, and shall,
shall have a weightage of one-third of in particular, provide free legal aid, by
the total votes cast in the meeting. suitable legislation or schemes or in
• The votes of all the state governments any other way, to ensure that
combined shall have a weightage of opportunities for securing justice are
two-thirds of the total votes cast in that not denied to any citizen by reason of
meeting. economic or other disabilities.Hence,
option b is correct.
Any act or proceeding of the Council will not
become invalid on the following grounds. • Option C is incorrect: Article 43- The
State shall endeavour to secure, by
• Any vacancy or deficit in the
suitable legislation or economic
constitution of the Council
organisation or in any other way, to all
• Any defect in the appointment of a workers, agricultural, industrial or
person as a member of the Council otherwise, work, a living wage,
• Any procedural irregularity of the conditions of work ensuring a decent
Council not affecting the merits of the standard of life and full enjoyment of
case leisure and social and cultural
opportunities and, in particular, the
State shall endeavour to promote
cottage industries on an individual or effectively preventing citizens from
cooperative basis in rural areas challenging laws relating to the
acquisition of pr
• Option D is incorrect: Article 43A-
The State shall take steps, by suitable • Option C is incorrect: The 42nd
legislation or in any other way, to Amendment changed the description
secure the participation of workers in of India from a "sovereign democratic
the management of undertakings, republic" to a "sovereign, socialist
establishments or other organisations secular democratic republic", and also
engaged in any industry. The article changed the words "unity of the
was inserted by the Constitution nation" to "unity and integrity of the
(Ninety-seventh Amendment) Act, nation".
2011.
• Option D is incorrect: 44th
Amendment Act, 1978 was passed by
32. Which constitutional amendment act made it the government in the year 1978 to
obligatory for the President to give assent to a revoke Some amendments made under
constitutional amendment bill? the 42-amendment act, 1976. The 44th
Amendment Act of 1978 was made to
A. 24th amendment act 1971
ensure that people also have the equal
B. 25th amendment act 1971 rights to decide the form of
C. 42nd amendment act 1976 government in which they will leave.
The 44th Amendment Act 1978 also
D. 44th amendment act 1978
safeguards people against the
Answer: A tendency of the majority to make rights
Explanation: in the future.

• Option A is correct: The 24 th

Constitutional Amendment Act of 1971 33. Which of the following Directive Principles of
made it obligatory for the President to State Policy in Part 4 of the Indian Constitution
give assent to a constitutional were added later by an amendment to the
amendment bill. constitution?

• Option B is incorrect: The 1. “To secure opportunities for the


Constitution (Twenty-Fifth healthy development of children”.
Amendment) Act 1971 was passed by 2. “To promote equal justice and to
Parliament on 20 April 1972. This provide free legal aid to the poor”.
aimed to reverse the Supreme Court’s
3. “Make provision for just and humane
ruling in R.C. Cooper v. Union of India
conditions of work and maternity
(1970), also popularly known as the
relief”.
bank nationalization case.
4. “Secure a living wage, a decent
o It amended Article 31 and
standard of living and social and
introduced a new Article 31C,
cultural opportunities for all workers”.
Select the correct answer using the codes given
below; 3. Parliament can amend any
Kesavananda part of the Constitution but it
A. 1 & 2 only
Bharati Case. cannot alter its “Basic
B. 1 & 4 only Structure’’.
C. 2 & 3 only
Which if the above pairs is/are matched correctly?
D. 3 & 4 only
A. 1 & 2
Answer: A
B. 2 & 3
Explanation:
C. 1 & 3
D. All of the above
Answer: B
Explanation:
• Pair 1 is incorrect: Champakam
Dorairajan Case (1952) - All
Fundamental Rights are superior over
DPSP. The Parliament can amend the
entire constitution including Part 3.
34. With reference to Supreme Court cases
relating relation between Part 3 and Part 4 of the • Pair 2 is correct: Golaknath case (1967)
constitution, consider the following pairs. - Fundamental Rights cannot be
abridged or diluted for the
Case Supreme Court observation implementation of DPSPs.
• Pair 3 is correct: Kesavananda Bharati
1. Fundamental Rights would Case (1973) - Parliament can amend
Champakam prevail over the DPSP in case any part of Constitution, but could not
Dorairajan of conflict between the two, destroy Basic Structure of the
case. and Fundamental Rights Constitution. The second clause of
cannot be amended by Article 31C was declared as
legislature in any case. unconstitutional and void as it was
against the Basic Structure of the
Constitution propounded in this case
2. Golaknath Fundamental Rights are
itself. However, the SC upheld the first
case. sacrosanct in nature and
provision of Article 31C. The court also
cannot be amended for
held that the power of Judicial review
implementation of DPSP.
cannot be taken out by Parliament.
35. Consider the following Constitutional members of Parliament.
Provisions:
• Rules of procedure in Parliament.
1. Admission or establishment of new
• Privileges of the Parliament, its
states.
members and its committees.
2. Election of President and its manner.
• Use of the English language in
3. Supreme Court and High Courts. Parliament.
4. Number of Puisne judges in the • Number of puisne judges in the
Supreme Court. Supreme Court.
5. Privileges of the Parliament, its • Conferment of more jurisdiction on the
members and its committees. Supreme Court.
Which of the above provisions can be amended • Citizenship-acquisition and
by a simple majority in the Parliament? termination.
A. 1, 2 & 3 only • Elections to Parliament and state
B. 1, 3 & 4 only legislatures.
C. 1, 4 & 5 only • Delimitation of constituencies.
D. 2, 4 & 5 only • Union territories
Answer: C • Fifth Schedule-administration of
Explanation: scheduled areas and scheduled tribes.

Statement 1, 4 and 5: These are amended through • Sixth Schedule-administration of tribal


simple majority. areas.

By Simple Majority of Parliament Statement 2 and 3: These are amended by special


majority of parliament.
• Admission or establishment of new
states. By Special Majority of Parliament and Consent of
States:
• Formation of new states and alteration
of areas, boundaries or names of • Election of the President and its
existing states. manner.

• Abolition or creation of legislative • Extent of the executive power of the


councils in states. Union and the states.

• Second Schedule-emoluments, • Supreme Court and high courts.

• Allowances, privileges and so on of the • Distribution of legislative powers


president, the governors, the Speakers, between
judges, etc. • the Union and the states.
• Quorum in Parliament. • Any of the lists in the Seventh
• Salaries and allowances of the Schedule.
• Representation of states in Parliament. provincial legislative councils to have
non-official majority.
• Power of Parliament to amend the
Constitution and its procedure (Article • Statement 3 is correct: The elected
368 itself). members were to be indirectly elected.
The local bodies were to elect an
electoral college, which in turn would
36. Consider the following statements with elect members of provincial
reference to features of the Indian Council
legislatures, who in turn would elect
Act of 1909:
members of the central legislature.
1. The number of members in the
Additional Features:
provincial legislative councils was
uniform. • It enlarged the deliberative functions
of the legislative councils at both the
2. The provincial legislative councils had
levels. For example, members were
an official majority.
allowed to ask supplementary
3. The elected members were to be questions, move resolutions on the
indirectly elected. budget, and so on.
Which of the statements given above is/are • It provided (for the first time) for the
correct? association of Indians with the
A. 1 only executive Councils of the Viceroy and
Governors. Satyendra Prasad Sinha
B. 2 and 3 only
became the first Indian to join the
C. 3 only Viceroy’s Executive Council. He was
D. 1 and 2 only appointed as the law member. Two
Indians were nominated to the
Answer: C
Council of the Secretary of State for
Explanation: Indian Affairs.
• Statement 1 is incorrect: It • It introduced a system of communal
considerably increased the size of the representation for Muslims by
legislative councils, both Central and accepting the concept of ‘separate
provincial. The number of members in electorate’. Under this, the Muslim
the Central Legislative Council was members were to be elected only by
raised from 16 to 60. The number of Muslim voters. Thus, the Act
members in the provincial legislative ‘legalized communalism’ and Lord
councils was not uniform. Minto came to be known as the Father
• Statement 2 is incorrect: The of Communal Electorate.
provincial legislative councils had • It also provided for the separate
official non majority. It retained representation of presidency
official majority in the Central corporations, chambers of commerce,
Legislative Council but allowed the universities and zamindars.
governor-generalship of Lord
37. Consider the following statements: Dalhousie.

1. Charter act of 1813 legalized the British B. It separated the legislative and
colonization of India executive functions of the Governor-
General’s council.
2. The charter act of 1833 ended the
monopoly of the East India Company C. It introduced an open competition
in India system of selection and recruitment of
civil servants.
Which of the statements given above is/are
correct? D. None of the above

A. 1 only Answer: D

B. 2 only Explanation:

C. Both 1 and 2 • Option A is correct: This Act was


passed when Lord Dalhousie was the
D. Neither 1 nor 2
Governor-General of India.
Answer: D
• Option B is correct: It separated, for
Explanation: the first time, the legislative and
• Statement 1 is incorrect: Charter act of executive functions of the Governor-
1813 ended the monopoly of the East General’s council.
India Company in India, the • Option C is correct: It introduced an
company’s monopoly in trade with open competition system of selection
China and trade in tea with India was and recruitment of civil servants. The
kept intact. covenanted civil service was thus
• Statement 2 is incorrect: The charter thrown open to the Indians also.
act of 1833 legalized the British Accordingly, the Macaulay
colonization of India. It ended the Committee (the Committee on the
activities of the East India Company Indian Civil Service) was appointed in
as a commercial body, it became an 1854.
administrative body. It provided that Additional information:
the company’s territories in India
Charter act of 1853: Other Features:
were held by the government ‘in trust
for His Majesty, His heirs and • The Charter Act of 1853 empowered
successors’. the British East India Company to
retain the territories and the revenues
in India in trust for the crown not for
38. Which of the following statements is any specified period, Unlike the
incorrect with reference to the Charter act of previous charter acts of 1793, 1813 and
1853? 1833 which renewed the charter for 20
A. This Act was passed during the years.
• This Act was passed when Lord the modern parliamentary form of
Dalhousie was the Governor-General government. The legislative wing of
of India. the Governor-General’s Council acted
as a parliament on the model of the
Governor-General’s office:
British Parliament.
• It provided for the addition of six new
members called legislative councilors
to the council {12 in total}. The 39. Consider the following statements with
Governor-General could nominate a reference to Pitt’s India Act 1784:

• vice president to the council and his 1. It recognized the political and
assent is required for all legislative administrative functions of the
actions. It established a separate Company
Governor-General’s legislative 2. It laid the foundations of central
council which came to be known as administration in India.
the Indian (Central) Legislative
3. It designated the Governor of Bengal
Council. This legislative wing of the
as the ‘Governor-General of Bengal’
council functioned as a mini-
Parliament, adopting the same Which of the statements given above is/are
procedures as the British Parliament. correct?

• The Law member (fourth member) A. 1 and 2 only


became a full member with the right B. 2 and 3 only
to vote.
C. 1 and 3 only
• The 12 members were – 1 Governor-
D. 1, 2 and 3
General, 1 Commander-in-Chief, 4
members of the Governor-General’s Answer: D
Council, 1 Chief Justice of the Explanation:
Supreme Court at Calcutta, 1 regular
• Statement 1 is correct: It recognized
judge of the Supreme Court at
the political and administrative
Calcutta, and 4 representative
functions of the Company
members drawn from
• Statement 2 is correct: It laid the
company’s servants with at least 10
foundations of central administration
years tenure, appointed by the local
in India.
governments of Bengal, Bombay,
Madras and North Western Provinces. • Statement 3 is correct: It designated
the Governor of Bengal as the
• The number of Board of Directors was
‘Governor-General of Bengal’
reduced from 24 to 18 out of which 6
people were to be nominated by the Additional Information:
British Crown. Pitt’s India Act 1784: Features
• This act served as the foundation of • It was the Pitts India Act which
distinguished between the 40. Which of the following Acts gave
commercial and political functions of recognition to the ‘portfolio system’ of Lord
the Company. Canning?

• It was the first step taken by the British A. Indian Councils Act 1861
Government to control and regulate B. Indian Councils Act 1892
the affairs of the East India Company
C. Indian Councils Act 1909
in India
D. Government of India Act 1919
• It designated the Governor of Bengal
as the ‘Governor-General of Bengal’ Answer: A
and created an Executive Council of Explanation:
four members to assist him. The first
• Indian Councils Act 1861 gave
such Governor-General was Lord
recognition to the ‘portfolio system’ of
Warren Hastings.
Lord Canning
• It provided for the establishment of a
• The Indian Councils Act 1861 was an
Supreme Court at Calcutta (1774)
Act of the Parliament of the United
comprising one chief justice and three
Kingdom that transformed India’s
other judges.
executive council to function as a
• It prohibited the servants of the cabinet run on the portfolio system.
Company from engaging in any
• It was introduced because the British
private trade or accepting presents or
Government wanted to involve the
bribes from the ‘natives’.
Indian people with the process of law
• It strengthened the control of the making. This Act was passed on 1st
British Government over the August 1861.
Company by requiring the Court of
• Main provisions of the Act:
Directors (governing body of the
Company) to report on its revenue, o It made a beginning of
civil, and military affairs in India. representative institutions by
associating Indians with law-
• It made the governors of Bombay and
making
Madras presidencies subordinate to
the Governor General of Bengal, o Viceroy nominated some Indians as
unlike earlier, when the three non-official members of his
presidencies were independent of one expanded council
another. ▪ Lord Canning nominated- Raja
• It was the Pitt's India Act which of Benaras, the maharaja of
distinguished between the Patiala and Sir Dinkar Rao
commercial and political functions of o Restored legislative making
the Company powers of Bombay and Madras
o Establishment of new Legislative
councils for Bengal, North-Western engagement of Indians with respect to
Frontier Province and Punjab the administration in British India.
o Viceroy could make provisions for • Statement 2 is correct: The act created
convenient transactions of business the stage for the development of
in the council. revolutionary forces in India
o It gave recognition to the ‘portfolio • Statement 3 is correct: It was the first
system’ of Lord Canning step towards the representative
o Ordinances could be issued by the government in modern India.
Viceroy without the concurrence of • Statement 4 is incorrect: Under India
the council during an emergency. Councils Act 1861 the Indian
However, the life of such an members were not eligible to oppose
ordinance was six months. any bill and most often the bills were
passed in one sitting without
discussion.
41. Consider the following statement with
reference to Indian Councils Act, 1892: Additional Information:

1. It increased the engagement of Indians Indian Councils Act 1892:


with respect to the administration in • The Indian Councils Act 1892 was an
British India. Act of British Parliament that
2. The act created the stage for the introduced various amendments to the
development of revolutionary forces in composition and function of legislative
India councils in British India.

3. It was the first step towards • Most notably, the act expanded the
representative government in modern number of members in the central and
India. provincial councils.

4. The Indian members were not eligible • Main provisions of the Act:
to oppose any bill o Increased non-official members in
Which of the statements given above is/are the council
correct? ▪ Bombay – 8
A. 1 and 2 only ▪ Madras – 20
B. 3 and 4 only ▪ Bengal – 20
C. 1, 2 and 3 only ▪ North-Western province -15
D. 4 only ▪ Oudh – 15
Answer: C ▪ Central Legislative Council
Explanation: minimum – 10, maximum 16

• Statement 1 is correct: It increased the o Members could now debate the


budget without having the ability to Answer: D
vote on it also barred from asking Explanation:
follow-up questions.
• The Government of India Act, 1935
o The Governor-General in Council derived material from four key
was given the authority to set rules sources viz. Report of the Simon
for member nomination, subject to Commission, discussions at the Third
the approval of the Secretary of State Round Table Conference, the White
for India. Paper of 1933 and the reports of the
o Made a limited and indirect Joint select committees.
provision for the use of election in
filling up non-official seats both in
43. With reference to the renunciation
central and provincial councils
of Indian citizenship, consider the
o Nomination for non-official following statements:
members to central legislative
1. An Indian Citizen of full age and
council (Bengal chamber of
capacity can renounce his Indian
commerce, governors for provincial
citizenship by making a declaration to
legislative council based on
that effect and having it registered.
recommendation of district boards,
municipalities, universities, trade 2. When a male person renounces
associations, zamindars and his citizenship every minor child of his
chambers) ceases to be an Indian citizen.
Which of the statements given above
is/are correct?
42. Government of India Act, 1935 derived
its material from which of the following A. 1 only
sources? B. 2 only
1. Report of the Simon Commission C. Both 1 and 2
2. Discussions at the Third Round Table D. Neither 1 nor 2
Conference
Answer: C
3. White Paper of 1933
Explanation:
4. Reports of the Joint select committees.
Statement 1 is correct: Declaration
Select the correct answer using the codes of Renunciation of Citizenship is made under
given below: Section 8 of the Citizenship Act, 1955: An
A. 1 and 2 only Indian Citizen of full age and capacity can
renounce his Indian citizenship by making a
B. 3 and 4 only
declaration to that effect and having it
C. 4 only registered. But
D. 1, 2, 3 and 4
if such a declaration is made during any war provided for Commonwealth
in which India is engaged, the registration Citizenship. But, this provision was
shall be withheld until the Central repealed by the
Government otherwise directs. Acquisition of Indian Citizenship
Statement 2 is correct: Where a person ceases
• The Citizenship Act of 1955
to be a citizen of India on renunciation of prescribes five ways of acquiring
citizenship under section 8 (1), every minor citizenship, viz, birth, descent,
child of that person shall there upon ceases to registration, naturalisation and
be a citizen of India However, such incorporation of territory:
child may, within one year after attaining
full age, can make a declaration in • Statement 1 is incorrect: i) By Birth
the prescribed form and the manner that A person born in India on or after
he wishes to resume Indian citizenship. January 26, 1950 but before July 1, 1987
is a citizen of India by birth
irrespective of the nationality of his
44. Consider the following parents. A person born in India on or
statements regarding to the Citizenship Act after July 1, 1987 is considered as a
(1955): citizen of India only if either of his
1. The children of foreign diplomats parents is a citizen of India at the time
posted in India and enemy aliens can of his birth. Further, those born in
acquire Indian citizenship by birth. India on or after December 3, 2004 are
considered citizens of India only if
2. According to the Citizenship Act of
both of their parents are citizens of
1955, from December 3, 2004 onwards,
India or one of whose parents is
a person born outside India shall not
a citizen of India and the other is not
be a citizen of India by descent.
an illegal migrant at the time of their
Which of the statements given above birth. The children of foreign
is/are correct? diplomats posted in India and enemy
A. 1 only aliens cannot acquire Indian
citizenship by birth.
B. 2 only
Acquisition of Indian Citizenship by
C. Both 1 and 2
Descent
D. Neither 1 nor 2
• Statement 2 is correct: A person
Answer: B born outside India on or after January
Explanation: 26, 1950 but before December 10, 1992
is a citizen of India by descent, if his
• The Citizenship Act (1955) provides
father was a citizen of India at the time
for the acquisition and loss of
of his birth. A person born outside
citizenship after the commencement of
India on or after December 10, 1992 is
the Indian Constitution. Originally,
considered as a citizen of India if
the Citizenship Act (1955) also
either of his parents is a citizen of Conferment of Rights on Overseas Citizen
India at the time of his birth. of India Cardholder

• December 3, 2004 onwards, a • An overseas citizen of


person born outside India shall not be India cardholder shall be entitled to
a citizen of India by descent, unless his such rights, as the Central
birth is registered at an Indian Government may specify on this
consulate within one year of the date behalf.
of birth or with the permission of the • An overseas citizen of
Central Government, after the expiry India cardholder shall not be entitled
of the said period. to the following rights (which are
delivered on a citizen of India)
45. Consider the following • OCI cardholder shall not be entitled to
statements regarding to the Overseas the right to equality of opportunity in
Citizen of India (OCI) Cardholder: matters of public employment.
1. OCI shall not be eligible for election as • He shall not be eligible for election as
Vice President. President.
2. OCI can be eligible for appointment as • He shall not be eligible for election as
a Judge of the High Court. Vice-President.
3. OCI shall not be eligible for being a • He shall not be eligible
member of the State Legislative for appointment as a Judge of
Assembly or the State Legislative the Supreme Court.
Council.
• He shall not be eligible
He can be entitled for registration as a voter. for appointment as a Judge of
Which of the statements given above the High Court.
is/are incorrect? • He shall not be entitled for registration
A. 2 and 4 only as a voter.

B. 1 and 3 only • He shall not be eligible for being a


member of the House of the People of
C. 2 and 4 only
the Council of States.
D. 1, 2 and 3
• He shall not be eligible for being a
Answer: A member of the State
Explanation: Legislative Assembly or the
The Citizenship (Amendment) Act, 2003, State Legislative Council.
made provision for acquisition of Overseas • He shall not be eligible
Citizenship of India (OCI) by the PIOs of for appointment to public
sixteen specified countries other than services and posts in connection
Pakistan and Bangladesh. with affairs of the Union or of
any State except for appointment services, emergency provisions and so
in such services and posts as on.
the Central Government
may specify. Statement 2 is correct: Originally
Hence option A is correct. (1949), the Constitution contained a
Preamble, 395 Articles (divided into 22
Parts) and 8 Schedules. Presently
46. Which of the following are the features (2019), it consists of a Preamble, about
of the Indian Constitution? 470 Articles (divided into 25 Parts) and
1. Federal System with Unitary Bias 12 Schedules. No other Constitution in
the world has so many Articles and
2. It is the lengthiest Written Constitution Schedules.
3. It provides for a Dual Citizenship
• Statement 3 is incorrect: Indian polity
4. Envisages an integrated Judiciary (Centre and states) provides for only a
Select the correct answer using the code single citizenship, that is, the Indian
given below. citizenship. In India, all citizens
irrespective of the state in which they
A. 1, 2 and 4 only
are born or reside enjoy the same
B. 2 and 4 only political and civil rights of citizenship
C. 1 and 3 only all over the country and no
discrimination is made between them.
D. 1, 2, 3 and 4
• Statement 4 is correct: The Indian
Answer: A
Constitution establishes a judicial
Explanation: Constitutionalism is specific system that is integrated as well as
limitations on general governmental powers independent. The Supreme Court
to prevent exercise of arbitrary decision- stands at the top of the integrated
making. In one word ‘Limited Governance’ is judicial system in the country. Below it,
the Constitutionalism, which is supposed to there are high courts at the state level.
reflect in the Constitutional Law of a Under a high court, there is a hierarchy
democratic state. of subordinate courts, that is, district
• Statement 1 is correct: The courts and other lower courts.
Constitution of India establishes a
federal system of Government. 47. Consider the following statements
However, the Indian Constitution also regarding the features of the Constituent
contains a large number of unitary or Assembly:
non-federal features, viz., a strong
1. It was totally an indirectly elected
Centre, single Constitution, single
body.
citizenship, flexibility of Constitution,
integrated judiciary, appointment of 2. Each Province and each Princely State
state governor by the Centre, all-India or group of States were allotted seats
proportional to their respective • The representatives from the Princely
population. states were to be elected by indirect
election by the members of the
3. The seats in each Province were
Provincial Legislative Assemblies that
distributed among the three main
had been established under the
communities - Muslims, Hindus and
Government of India Act, 1935. While
General.
the method of selection in the case of
4. Members of each community in the representatives of Princely States was
Provincial Legislative Assembly to be determined by consultation. In
elected their own representatives by most of the princely states, the
the method of proportional representatives were nominated by the
representation with single transferable heads of the princely states.
vote
• Statement 2 is correct: Each province
Which of the above statements are correct? and princely state (or group of states in
A. 1 and 3 only case of small states) were to be allotted
seats in proportion to their respective
B. 2, 3 and 4 only
population. Roughly, one seat was to
C. 1, 3 and 4 only be allotted for every million
D. 2 and 4 only population.

Answer: D • Statement 3 is correct: Seats allocated


to each British province were to be
Explanation:
divided among the three principal
• The Constituent Assembly was communities–Muslims, Sikhs and
expected to work within the General (all except Muslims and
framework of the Cabinet Mission Sikhs), in proportion to their
scheme alone. However, these population.
limitations were removed by the
Statement 4 is correct: Members of each
Indian Independence Act, 1947 under
community in the Provincial Legislative
which it was made free to frame any
Assembly elected their own representatives
constitution it pleased.
by the method of proportional representation
• Statement 1 is incorrect: The total with single transferable vote.
strength of the Constituent Assembly
was to be 389. Of these, 296 seats were
to be allotted to British India and 93 48. Which of the following functions were
seats to the princely states. Out of 296 performed by the Constituent Assembly of
seats allotted to the British India, 292 India?
members were to be drawn from the 1. It passed the ‘Indian Independence
eleven governors’ provinces and four Act’.
from the four Chief Commissioners’
2. It terminated India’s membership of
provinces, one from each.
the Commonwealth.
3. It adopted the national flag. • Statement 4 is correct: It adopted the
national song on January 24, 1950.
4. It adopted the national song.
Select the correct answer using the code • Additional Information: Other
given below. functions performed by it were:

A. 1 only • It adopted the national anthem on


January 24, 1950.
B. 3 and 4 only
• It elected Dr. Rajendra Prasad as the
C. 1, 2 and 4 only
first President of India on January 24,
D. 2 and 3 only 1950.
Answer: B
Explanation: 49. Consider the following pairs of various
• The Constituent Assembly held its first committees of the Constituent Assembly and
their chairman:
meeting on December 9, 1946. The
Muslim League boycotted the meeting Name of the
and insisted on a separate state of Chairman
Committees
Pakistan. The meeting was, thus,
attended by only 211 members. In Union Constitution Dr. B.R.
addition to the making of the Committee Ambedkar
Constitution and enacting of ordinary
laws, the Constituent Assembly also Sardar
Provincial Constitution
performed many other functions. Vallabhbhai
Committee
Patel
• Statement 1 is incorrect: On 1st July,
1947, the British Parliament passed the Advisory Committee on
‘Indian Independence Act’, to divide Fundamental Rights, Jawaharlal
British India into India and Pakistan. Minorities and Tribal Nehru
As a consequence of the Partition and Excluded Areas
under the plan of 3 June 1947 those
members who were elected from Which of the above pairs is/are correctly
territories which fell under Pakistan matched?
ceased to be members of the A. 1 and 3 only
Constituent Assembly. The number of
B. 2 only
members in the Assembly was reduced
to 299. C. 2 and 3 only

• Statement 2 is incorrect: It ratified the D. 1, 2 and 3


India’s membership of the Answer: B
Commonwealth in May 1949
Explanation: The Constituent Assembly
• Statement 3 is correct: It adopted the appointed a number of committees to deal
national flag on July 22, 1947. with different tasks of constitution-making.
Out of these, eight were major committees Explanation:
and the others were minor committees. According to Dr. B.R. Ambedkar, the
• Pair 1 is incorrect: Union Constitution Directive Principles of State Policy is a ‘novel
Committee was headed by Jawaharlal feature’ of the Indian Constitution. They are
Nehru. enumerated in Part IV of the Constitution.
• Pair 2 is correct: Provincial Additional Information:
Constitution Committee was headed • The Directive Principles are meant for
by Sardar Vallabhbhai Patel. promoting the ideal of social and
• Pair 3 is incorrect: Advisory economic democracy. They seek to
Committee on Fundamental Rights, establish a ‘welfare state’ in India.
Minorities and Tribal and Excluded However, unlike the Fundamental
Areas was headed by Sardar Rights, the directives are non-
Vallabhbhai Patel. justiciable in nature, that is, they are
not enforceable by the courts for their
Additional Information: Other committees
violation.
were:
• They impose a moral obligation on the
• Union Powers Committee - Jawaharlal
state authorities for their application.
Nehru
But, the real force (sanction) behind
• Drafting Committee - Dr. B.R. them is political, that is, public
Ambedkar opinion.
• Rules of Procedure Committee - Dr. • B.R. Ambedkar said in the Constituent
Rajendra Prasad Assembly that ‘a government which
• States Committee (Committee for rests on popular vote can hardly ignore
Negotiating with States) - Jawaharlal the Directive Principles while shaping
Nehru its policy. If any government ignores
them, it will certainly have to answer
• Steering Committee - Dr. Rajendra
for that before the electorate at the
Prasad
election time’

50. Which of the following was regarded as


51. Consider the following statements
the ‘novel feature’ of Indian Constitution by
regarding the idea of popular sovereignty:
Dr. BR Ambedkar?
1. It means that the authority of a state
A. Constitutional Remedies
and its government are created and
B. Provisions of reservation in sustained by the consent of its people.
Constitution itself
2. Indian Democracy is based on the idea
C. Directive Principles of State Policy of popular sovereignty.
D. Preamble Which of the above statements is/are
Answer: C correct?
A. 1 only
B. 2 only 52. A constitutional government by
definition is a [UPSC 2020]
C. Both 1 and 2
A. government by legislature
D. Neither 1 nor 2
B. popular government
Answer: C
C. multi-party government
Explanation:
D. limited government
• Statement 1 is correct: Popular
sovereignty is the principle that the Answer: D
authority of a state and its government Explanation:
are created and sustained by the
A constitutional government is the one that is
consent of its people, through their
defined by the constitution of the country.
elected representatives (rule by the
Constitution of a country is supreme law of
people), who are the source of all
the land, it is empowered with the sovereign
political power.
authority of the people (we the people) by the
• It is closely associated with social framers and the consent of the legislatures of
contract philosophers such as Thomas the states, it is the source of all government
Hobbes, John Locke and Jean-Jacques powers, and also provides important
Rousseau. limitations on the government that protect the
• Statement 2 is correct: Democracy is of fundamental rights of citizens.
two types–direct and indirect. In direct
democracy, the people exercise their
53. Consider the following objectives:
supreme power directly. In indirect
democracy, on the other hand, the 1. Liberty of Faith
representatives elected by the people 2. Dignity of the nation
exercise the supreme power and thus
3. Equality of Opportunity
carry on the government and make the
laws. 4. Political justices
• The Indian Constitution provides for Which of the above-given objectives are
representative parliamentary stated in the Preamble to the Constitution of
democracy under which the executive India?
is responsible to the legislature for all A. 1, 2 and 4 only
its policies and actions. Indian
B. 1, 3 and 4 only
democratic polity, as stipulated in the
Preamble, is based on the doctrine of C. 2 and 4 only
popular sovereignty, that is, D. 1, 2, 3 and 4 only
possession of supreme power by the
Answer: B
people.
Explanation:
The Preamble in its present form reads: B. 2 and 4 only
• “We, THE PEOPLE OF INDIA, having C. 2 and 3 only
solemnly resolved to constitute India D. 2, 3 and 4 only
into a SOVEREIGN SOCIALIST
SECULAR DEMOCRATIC REPUBLIC Answer: D
IN and to secure to all its citizens: Explanation:
• JUSTICE, Social, Economic and • Option 1 is incorrect: Fundamental
Political; Rights are justiciable and the
aggrieved person can move to the
• LIBERTY of thought, expression,
courts for the violation of his/her
belief, faith and worship;
rights. They are defended by Supreme
• EQUALITY of status and of Court and the aggrieved person can
opportunity; and to promote among move directly to Supreme Court.
them all;
• Option 2 is correct: The Preamble is
• FRATERNITY assuring the dignity of neither a source of power to legislature
the individual and the unity and nor a prohibition upon the powers of
integrity of the Nation; legislature and is non-justiciable, that
• OUR CONSTITUENT ASSEMBLY this is, its provisions are not enforceable in
twenty-sixth day of November, 1949, courts of law.
do HEREBY • Option 3 is correct: Directive
ADOPT, ENACT AND GIVE TO Principles of State Policy are non
OURSELVES THIS CONSTITUTION”. justiciable in nature. They are not
legally enforceable by the courts for
Dignity of the Nation is not mentioned in the
their violation. This means that the
Preamble.
government cannot be compelled to
Hence option B is the correct answer. implement them. However, Article 37
says that these principles are
fundamental in the governance of the
54. Which of the following parts of the
country and the State shall apply these
constitution is/are non-justiciable by
while making laws.
the courts?
• Option 4 is correct: Fundamental
1. Fundamental Rights
Duties are non-justiciable in nature as
2. Preamble there is no legal sanction against their
3. Directive Principles of State Policy violation. Moreover, the constitution
does not provide for their direct
4. Fundamental duties
enforcement by the courts. However,
Select the correct answer using the Parliament is free to enforce them by
code given below. suitable legislation.
A. 1, 2 and 3 only
55. Which of the following words was/were A. 1 and 2 only
added to the Preamble of the Indian
B. 2 and 3 only
Constitution by the 42nd Constitutional
Amendment Act, 1976? C. 1 and 3 only

1. Fraternity D. 1, 2 and 3

2. Equality Answer: A

3. Republic Explanation:

4. Socialist • Article 22 grants protection to persons


who are arrested or detained.
5. Secular
Detention is of two types, namely,
Select the correct answer using the code punitive and preventive. Punitive
given below. detention is to punish a person for an
A. 1, 2 and 5 only offense committed by him after trial
and conviction in a court. Preventive
B. 1, 3 and 4 only
detention, on the other hand, means
C. 4 and 5 only detention of a person without trial and
D. 1, 2, 4 and 5 only conviction by a court. Its purpose is not
to punish a person for a past offence
Answer: C
but to prevent him from committing an
Explanation: offence in the near future. Thus,
Preamble has been amended by the 42nd preventive detention is only a
Constitutional Amendment Act of 1976, precautionary measure and based on
which added three new words—socialist, suspicion.
secular and integrity. • Article 22 has two parts–the first part
deals with the cases of ordinary law
and the second part deals with the
56. Consider the following with reference to
cases of preventive detention law.
protection against preventive detention in
The second part of Article 22 grants
the constitution.
protection to persons who are arrested
1. It is available to both citizens as well as or detained under a preventive
aliens. detention law. This protection is
2. The grounds for detention shall available to both citizens as well as
invariably be communicated with the aliens and includes the following:
person detained. • The detention of a person cannot
3. The detention cannot exceed 3 months exceed three months unless the
unless extended by the advisory board. advisory board reports sufficient cause
for extended detention. The board is to
Which of the statements given above is/are
consist of judges of a high court.
correct?
Hence, statement 3 is correct.
• The grounds of detention should be foreign affairs and the security of
communicated to the deten However, India. Both the Parliament as well as
the facts considered to be against the the state legislatures can concurrently
public interest need not be disclosed. make a law of preventive detention for
Hence, statement 2 is incorrect. reasons connected with the security of
a state, the maintenance of public order
• The detenu should be afforded an
and the maintenance of supplies and
opportunity to make representation
services essential to the community.
against the detention order.
Additional information:
Preventive Detention 57. Consider the following with reference to
Prohibition of child labour in India.
• Article 22 also authorizes the
1. Article 24 prohibits employment of
Parliament to prescribethe
children below the age of 14 in any
circumstances and the classes of cases
occupation.
in which a person can be detained for
more than three months under a 2. The article also provides for the
preventive detention law without establishment of the National
obtaining the opinion of an advisory Commission for Protection of Child
board; Rights (NCPCR).
• the maximum period for which a Which of the statements given above is/are
person can be detained in any classes correct?
of cases under a preventive detention A. 1 only
law; and
B. 2 only
the procedure to be followed by an
advisory board in an inquiry. C. Both 1 and 2
• The 44th Amendment Act of 1978 has D. Neither 1 nor 2
reduced the period of detention Answer: D
without obtaining the opinion of an
Explanation:
advisory board from three to two
months. However, this provision has • Article 24 prohibits the employment of
not yet been brought into force, hence, children below the age of 14 years in
the original period of three months still any factory, mine or other hazardous
continues. activities like construction work or
railway. But it does not prohibit their
• The Constitution has divided the
employment in any harmless or
legislative power with regard to
innocent work. Hence, statement 1 is
preventive detention between the
incorrect.
Parliament and the state legislatures.
The Parliament has exclusive authority • The Commissions for Protection of
to make a law of preventive detention Child Rights Act, 2005 was enacted to
for reasons connected with defence, provide for the establishment of a
National Commission and State • Article 25 says that all persons are
Commissions for Protection of Child equally entitled to freedom of
Rights and Children’s Courts for conscience and the right to freely
providing speedy trial of offences profess, practice and propagate
against children or of violation of child religion. Hence, statement 1 is correct.
rights. Hence, statement 2 is incorrect. • However, these rights are subject to
• In 2006, the government banned the public order, morality, health and
employment of children as domestic other provisions relating to
servants or workers in business fundamental rights. Hence, statement
establishments like hotels, dhabas, 2 is correct.
restaurants, shops, factories, resorts, • Further, the State is permitted
spas, tea-shops and so on. It warned to: regulate or restrict any economic,
that anyone employing children below financial, political or other secular
14 years of age would be liable for activity associated with religious
prosecution and penal action. practice; and provide for social welfare
and reform or throw open Hindu
58. Consider the following with reference to religious institutions of a public
Indian secularism. character to all classes and sections of
Hindus.
1. The Indian constitution provides for
freedom of conscience. • Hence, statement 3 is correct.

2. The freedom of conscience of a person • Also, this does not include a right to
can be restricted on the basis that it convert another person to one’s own
goes against public morality. religion. Forcible conversions impinge
on the ‘freedom of conscience’
3. The state can make a law to regulate guaranteed to all the persons alike.
any secular activity associated with
religious practice related to such a
conscience. 59. Which of the following is an accurate
difference between rights under Article 25
Which of the statements given above is/are
and Article 26?
correct?
1. Article 25 protects individual religious
A. 1 and 2 only
rights while Article 26 protects
B. 2 and 3 only collective religious rights.
C. 1 and 3 only 2. Rights under Article 25 can be
D. 1, 2 and 3 restricted on the basis of public
morality while that is not the case for
Answer: D
Article 26.
Explanation:
Which of the statements given above is/are
correct?
A. 1 only 1. It is a constitutional right.
B. 2 only 2. It is part of the basic structure of the
constitution.
C. Both 1 and 2
3. Article 31B can be said to be against the
D. Neither 1 nor 2
right of private property.
Answer: A
Which of the statements given above is/are
Explanation: correct?
• Article 25 says that all persons are A. 1 and 2 only
equally entitled to freedom of
B. 2 and 3 only
conscience and the right to freely
profess, practice and propagate C. 1 and 3 only
religion. D. 1, 2 and 3
According to Article 26, every religious
denomination or any of its section shall Answer: C
have the following rights:Right to Explanation:
establish and maintain institutions for • Originally, the right to property was
religious and charitable purposes; one of the seven fundamental rights
• Right to manage its own affairs in under Part III of the Constitution. It
matters of religion; was dealt by Article 19(1) (f) and
Article 31.
• Right to own and acquire movable and
Since the commencement of the
immovable property; and
Constitution, the Fundamental Right
• Right to administer such property in to Property has been the most
accordance with law. controversial. It has caused
• Article 25 guarantees rights of individuals, confrontations between the Supreme
while Article 26 guarantees rights of Court and the Parliament. It has led to
religious denominations or their sections. a number of Constitutional
In other words, Article 26 protects amendments, that is, 1st, 4th, 7th, 25th,
collective freedom of religion. Hence, 39th, 40th and 42nd Amendments.
statement 1 is correct. • Through these amendments, Articles
• Like the rights under Article 25, the rights 31A, 31B and 31C have been added and
under Article 26 are also subject to public modified from time to time to nullify
order, morality and health but not subject the effect of Supreme Court
to other provisions relating to the judgements and to protect certain laws
Fundamental Rights. Hence, statement 2 is from being challenged on the grounds
incorrect. of contravention of Fundamental
Rights (i.e. right to property). Most of
the litigation centered around the
60. Consider the following with reference to obligation of the state to pay
right to property in the Indian Constitution. compensation for acquisition or
requisition of private property. Hence, C. 1-iii, 2-ii, 3-i
statement 3 is correct. D. 1-iii, 2-i, 3-ii
• Therefore, the 44th Amendment Act of Answer: B
1978 abolished the right to property as
a Fundamental Right by repealing Explanation:
Article 19(1) (f) and Article 31 from EXCEPTIONS TO FUNDAMENTAL
Part III. Instead, the Act inserted a new RIGHTS
Article 300A in Part XII under the • Saving of Laws Providing for
heading ‘Right to Property’. It Acquisition of Estates, etc. Article
provides that no person shall be 31A16 saves five categories of laws
deprived of his property except by from being challenged and invalidated
authority of law. Thus, the right to on the ground of contravention of the
property still remains a legal right or a fundamental rights conferred by
constitutional right, though no longer Article 14 (equality before law and
a fundamental right. Hence, statement equal protection of laws) and Article 19
1 is correct. (protection of six rights in respect of
• It is not a part of the basic structure of speech, assembly, movement, etc.).
the Constitution. Hence, statement 2 is They are related to agricultural land
incorrect. reforms, industry and commerce and
include the following:
• Acquisition of estates17 and related
61. Match the following.
rights by the State;
Taking over the management of
Article Purpose
properties by the State;
Amalgamation of corporations;
1. Article i. Saving Laws added in 9th Extinguishment or modification of
31A schedule rights of directors or shareholders of
corporations; and
2. Article ii. Saving Laws providing • Extinguishment or modification of
31B for acquisition of estates mining leases.
Validation of Certain Acts and
Regulations Article 31B saves the acts
3. Article iii. Saving laws giving effect
and regulations included in the Ninth
31C to directive principles in the
Schedule18 from being challenged and
constitution
invalidated on the ground of
contravention of any of the
Choose the correct answer
fundamental rights. Thus, the scope of
A. 1-i, 2-ii, 3-iii Article 31B is wider than Article 31A.
B. 1-ii, 2-i, 3-iii Article 31B immunizes any law
included in the Ninth Schedule from
all the fundamental rights whether or • No Social and Economic Rights The list
not the law falls under any of the five is not comprehensive as it mainly
categories specified in Article 31A. consists of political rights. It makes no
provision for important social and
• Saving of Laws Giving Effect to
economic rights like right to social
Certain Directive PrinciplesArticle
security, right to work, right to
31C, as inserted by the 25th
employment, right to rest and leisure
Amendment Act of 1971, contained the
and so on
following two provisions:
No Clarity: They are stated in a vague,
No law that seeks to implement the
indefinite and ambiguous manner. The
socialistic directive principles specified
various phrases and words used in the
in Article 39(b) or c) shall be void on
chapter like ‘public order’, ‘minorities’,
the ground of contravention of the
‘reasonable restriction’, ‘public
fundamental rights conferred by
interest’ and so on are not clearly
Article 14 (equality before law and
defined.
equal protection of laws) or Article 19
(protection of six rights in respect of • No Permanency: They are not
speech, assembly, movement, etc.) sacrosanct or immutable as the
Hence, option B is the correct one. Parliament can curtail or abolish them,
as for example, the abolition of the
fundamental right to property in 1978.
62. Which of the following is not a criticism Suspension During Emergency: The
generally done of fundamental rights in the suspension of their enforcement
Indian constitution? during the operation of National
A. Excessive Limitations Emergency (except Articles 20 and 21)
is another blot on the efficacy of these
B. Broad scope
rights.
C. Suspension during Emergency
• Expensive Remedy: The judiciary has
D. Expensive Remedy been made responsible for defending
Answer: B and protecting these rights against the
interference of the legislatures and
Explanation:
executives.
• The Fundamental Rights enshrined in Preventive Detention: The critics assert
Part III of the Constitution have met that the provision for preventive
with a wide and varied criticism. The detention (Article 22) takes away the
arguments of the critics are spirit and substance of the chapter on
Excessive Limitations: They are fundamental rights.
subjected to innumerable exceptions,
• No Consistent: Philosophy According
restrictions, qualifications and
to some critics, the chapter on
explanations.
fundamental rights is not the product
of any philosophical principle
Hence, option B is the correct one. nullity. It is the very soul of the
Constitution and the very heart of it’.
The Supreme Court has ruled that
63. Consider the following with reference to Article 32 is a basic feature of the
the right to constitutional remedies under
Constitution. Hence, it cannot be
article 32.
abridged or taken away even by way
1. It empowers supreme court and high of an amendment to the Constitution.
court to issue writs for enforcement of
• It contains the following four
the fundamental rights.
provisions:
2. The right to move to any court for the
o The right to move the Supreme
enforcement of fundamental rights can
Court by appropriate
be suspended by the president during
proceedings for the
a national emergency.
enforcement of the
Which of the statements given above is/are Fundamental Rights is
correct? guaranteed. Hence, statement 1
A. 1 only is incorrect. The right to move
the High court for enforcement
B. 2 only
of fundamental rights is
C. Both 1 and 2 guaranteed under Article 226.
D. Neither 1 nor 2 • The Supreme Court shall have power
Answer: C to issue directions or orders or writs for
the enforcement of any of the
Explanation:
fundamental rights. The writs issued
• A mere declaration of fundamental may include habeas corpus,
rights in the Constitution is mandamus, prohibition, certiorari and
meaningless, useless and worthless quo-warranto.
without providing an effective
• Parliament can empower any other
machinery for their enforcement, if
court to issue directions, orders and
and when they are violated.
writs of all kinds. However, this can be
• Hence, Article 32 confers the right to done without prejudice to the above
remedies for the enforcement of the powers conferred on the Supreme
fundamental rights of an aggrieved Court. Any other court here does not
citizen. In other words, the right to get include high courts because Article 226
the Fundamental Rights protected is in has already conferred these powers on
itself a fundamental right. This makes the high courts. (d) The right to move
the fundamental rights real. That is the Supreme Court shall not be
why Dr. Ambedkar called Article 32 as suspended except as otherwise
the most important article of the provided for by the Constitution. Thus
Constitution – ‘an article without the Constitution provides that the
which this constitution would be a President can suspend the right to
move any court for the enforcement of 1. Fundamental Rights are absolute
fundamental rights during a national 2. Positive FRs entail limitations on the
emergency (Article 359). Hence, government
statement 2 is correct.
3. FRs are justiciable in nature
Which of the statements given above is/are
64. The Constitution of India provides for correct?
six Fundamental Rights. Consider the
following FRs and the associated articles: A. 1 and 2 only

Which of the pairs given above is/are B. 2 only


correctly matched? C. 1 and 3 only
A. 1 and 2 only D. 3 only
B. 3 and 4 only Answer: D
C. 1 and 3 only Explanation:
D. 2 and 4 only • Fundamental rights are different from
Answer: A ordinary legal rights in the manner in
which they are enforced. If a legal right
Explanation:
is violated, the aggrieved person
• The Fundamental Rights: The cannot directly approach the SC
Constitution of India provides for six bypassing the lower courts. He or she
Fundamental Rights: should first approach the lower courts.
o Right to equality (Articles 14– • Some of the fundamental rights are
18) available to all citizens while the rest
o Right to freedom (Articles 19– are for all persons (citizens and
22) foreigners).
Fundamental rights are not absolute
o Right against exploitation
rights. They have reasonable
(Articles 23–24)
restrictions, which means they are
o Right to freedom of religion subject to the conditions of state
(Articles 25–28) security, public morality and decency
o Cultural and educational rights and friendly relations with foreign
(Articles 29–30) countries.

o Right to constitutional remedies • They are justiciable, implying they are


(Article 32) enforceable by courts. People can
approach the SC directly in case of
violation of fundamental rights.
65. Consider the following statements
• Fundamental rights can be amended
regarding characteristics of Fundamental
by the Parliament by a constitutional
Rights:
amendment but only if the amendment
does not alter the basic structure of the Answer: D
Constitution. Explanation:
• Fundamental rights can be suspended • According to Article 12 of the
during a national emergency. But, the Constitution, State includes the
rights guaranteed under Articles 20 following:
and 21 cannot be suspended. The Government and Parliament of
The application of fundamental rights India (both executive and legislative
can be restricted in an area that has part of the Union);
been placed under martial law or
• The Government and legislative of
military rule.
states (both executive and legislative of
Statement 1 is incorrect: Fundamental
each of the state);
Rights are not absolute but qualified.
Reasonable restrictions can be • All local authorities within the
imposed on FRs. The reasonability of territory of India (municipalities,
such restrictions is decided by the SC. panchayats, district boards,
improvement trusts, etc.); and
• Statement 2 is incorrect: Some FRs is
negative in character while others are • All other authorities under the control
positive. Negative FRs entail of the Government of India (both
limitations on the government, while statutory or non-statutory authorities
positive FRs imposes an obligation on like LIC, ONGC, SAIL, etc.).
the government to take measures.
• Statement 3 is correct: FRs are 67. Consider the following statements with
justiciable in nature reference to Article 22:
1. Punitive and Preventive detention
66. According to Article 12 of the provisions are applicable to both
Constitution, State includes which of the citizens and aliens
following?
2. Legislative power with regard to
1. Both executive and legislative part of preventive detention exclusively for
the Union and of each state defence, foreign affairs and security of
2. District boards India is with the Parliament.

3. Improvement trusts 3. Preventive detention is applicable to


those acts which are of a criminal or
Select the correct code: quasi-criminal nature
A. 1 and 2 only Which of the statements given above is/are
B. 2 and 3 only correct?
C. 1 and 3 only A. 1 only

D. 1,2 and 3 B. 2 and 3 only


C. 3 only preventive detention between
Parliament (exclusively for defence,
D. 1 and 2 only
foreign affairs and security of India)
Answer: B and the state legislatures
Explanation:
• Statement 1 is incorrect: Preventive
• Article 22: Protection against arrest detection provisions are applicable to
and detention: Two types of detention: both citizens as well as aliens
punitive and preventive
• Statement 2 is correct: Legislative
• First part of article 22Right to be power with regard to preventive
informed on the grounds of arrest detention exclusively for defence,
foreign affairs and security of India is
• Right to consult and be defended by a
with the Parliament.
legal practitioner
• Statement 3 is correct: Preventive
• Right to be produced before a
detention is applicable to those acts
magistrate within 24 hours
which are of a criminal or quasi-
• Right to be released after 24 hours criminal nature.
unless the magistrate authorizes
further detention
68. Consider the following with reference to
• These are applicable to those acts
Regulating Act, 1773:
which are of a criminal or quasi-
criminal nature 1. The act provided for the appointment
of a Governor-General along with four
• These safeguards are not available to a
Councillors.
person detained under preventive
detention law and also to an alien 2. A Supreme Court of Judicature was to
be established at Calcutta with Sir
• Preventive detection provisions are
Elijah Impey as the first Chief Justice.
applicable to both citizens as well as
aliens Detention of a person cannot 3. Governors-in-Council of Bombay and
exceed three months unless an Madras were required to pay due
advisory board recommends the same obedience to the orders of the
(the board is to consist of judges of a Governor - General of Bengal.
high court) Which of the above statements is/are correct?
• Ground of detention should be A. 1 and 2 only
communicated to Detenu. However,
B. 1 and 3 only
facts considered important may not be
communicated Opportunity for C. 1, 2 and 3
representation should be provided. D. 2 and 3 only
• Constitution has divided the Answer: C
legislative power with regard to
• Statement 1 is correct as the act
provided for the appointment of a
Governor-General along with four 70. Consider the following statements with
Councillors. respect to Right to Constitutional Remedies:
• Statement 2 is correct as a Supreme 1. The right to move the Supreme Court
Court of Judicature was to be under this is guaranteed by the
established at Calcutta with Sir Elijah Constitution.
Impey as the first Chief Justice.
2. It is said to be the very soul of the
• Statement 3 is correct as Governors-in- Constitution.
Council of Bombay and Madras were
3. This remedy is mentioned under the
required to pay due obedience to the
Article 32 of Indian Constitution.
orders of Governor -General of Bengal.
Which of the above statements is/are correct?
A. 1 and 2
69. Consider the following statements with
respect to Kesavananda Bharati case: B. 2 and 3
1. It held that the preamble can be C. 1 and 3
amended. D. 1, 2 and 3
2. It was subject to the condition that no Answer: D
amendment is done to the Basic
All of the above statements are correct.
Features.
Additional Information
Which of the statements given above is/are
correct? Part III of the Constitution provides for legal
remedies for the protection of these rights
A. 1 only
against their violation by the State or other
B. 2 only institutions/individuals. It entitles the
C. Both 1 and 2 citizens of India to move the Supreme Court
or High Courts for the enforcement of these
D. Neither 1 nor 2
rights. The State is forbidden from making
Answer: C any law that may conflict with the
In the historic case of Kesavananda Bharati (1973), Fundamental Rights.
the Supreme Court held that the Preamble can be
amended, subject to the condition that no
71. Consider the following statements with
amendment is done to the basic features ‘. In other
respect to the Writ of Certiorari:
words, the Court held that the basic elements or the
fundamental features of the Constitution as 1. It is issued by a lower court to a higher
contained in the Preamble cannot be altered by an court.
amendment under Article 368 The Court stated 2. Like prohibition, it is both preventive
that the opinion tendered by it in the Berubari as well as curative.
Union (1960) in this regard was wrong.
3. It is not available against legislative
bodies and private individuals or advertisement.
bodies. 3. Right to know about Government
Which of the above statements is/are correct? activities.
A. 1 and 2 4. Right against imposition of pre
B. 2 only censorship on a newspaper.

C. 3 only Choose the correct answer using the codes given


below:
D. 2 and 3
A. 1, 2 and 3
Answer: C
B. 2, 3 and 4
• Certiorari In the literal sense, it means
C. 1, 3 and 4
‘to be certified’ or ‘to be informed’.
D. 1, 2, 3 and 4
• It is issued by a higher court to a lower
court or tribunal either to transfer a Answer: D
case pending with the latter to itself or All of the above are included under the right of
to squash the order of the latter in a speech and expression.
case.
It is issued on the grounds of excess of
jurisdiction or lack of jurisdiction or 73. Consider the following with regard to
error of law. Thus, unlike prohibition, Fundamental Rights:
which is only preventive, certiorari is 1. President, on the advice of the council
both preventive as well as curative. of ministers, can impose reasonable
• Previously, the writ of certiorari could restrictions on fundamental rights of
be issued only against judicial and the citizens.
quasi-judicial authorities and not 2. Judiciary decides upon the
against administrative authories. reasonableness of the restrictions
However, in 1991, the Supreme Court placed on fundamental rights of the
ruled that the certiorari can be issued citizens.
even against administrative authorities
Which of the above statements is/are correct?
affecting rights of individuals. Like
prohibition, certiorari is also not A. 1 only
available against— legislative bodies B. 2 only
and private individuals or bodies.
C. Both 1 and 2
D. Neither 1 nor 2
72. Which of the following fundamental rights
Answer: B
are included under the Right to Speech and
Expression: Parliament can impose reasonable restrictions on
fundamental rights of the citizens. Judiciary
1. Freedom of press
2. Freedom of commercial
decides upon the reasonableness of the restrictions Answer: A
placed on fundamental rights of the citizens Explanation:
Statement A is correct: Recognition of personality
of every individual as sacred in a democratic setup.
74. Which Article of the Constitution of India
safeguards one’s right to marry the person of Additional Info:
one’s choice? According to K.M Munshi, the phrase 'dignity of
A. Article 19 the individual' signifies that the Constitution not
only ensures material betterment and maintains a
B. Article 21
democratic set-up, but that it also recognises that
C. Article 25 the personality of every individual is sacred. So,
D. Article 29A Statement 1 is correct.
Answer: B The Preamble and Article 38 of the Constitution
envision social justice as the arch to ensure life to
Article 21 is at the heart of the Constitution. It is our
be meaningful and liveable with human dignity.
living Constitution's most organic and progressive
provision. It can only be invoked when a person's
'life or 'personal liberty' is taken away by the 'State,' 76. Consider the following statements with
as defined in Article 12. As a result, private reference to Fundamental Duties:
individuals' violations of the right are not covered
1. It was added by the 42nd
by Article 21. Two rights are guaranteed under it
Constitutional Amendment Act
i.e., the right to life and the right to personal liberty.
adding Part IV to the Constitution.
2. It was inspired by the Constitution of
75. Which of the following statements best erstwhile USSR, the necessity of which
describe the phrase "Dignity of Individual", was felt during the emergency of 1975-
enshrined in the Preamble? 77.
A. Recognition of the personality of every Which of the statements given above is/are
individual as sacred in a democratic correct?
setup.
A. 1 only
B. Duty of every citizen to promote
B. 2 only
harmony and the spirit of common
brotherhood. C. Both 1 and 2
C. Overcoming hindrances to national D. Neither 1 nor 2
integration like communalism, Answer: B
regionalism, casteism, secessionism
Explanation:
etc.
Statement 1 is incorrect: The Fundamental
D. Securing all citizens equality of status
Duties are dealt with Article 51A under Part-
and opportunity in the political,
IV A of the Indian Constitution.
economic and social arena.
Additional Info: collect taxes for the promotion or
The Constitution of erstwhile USSR inspires maintenance of all religions.
the Fundamental Duties in the Indian Select the correct answer using the code given
Constitution. below:
Notably, none of the Constitutions of major A. 1 only
democratic countries like USA, Canada, B. 2 and 3 only
France, Germany, Australia and so on
specifically contain a list of citizens' duties. C. 1 and 3 only
Perhaps, the Japanese Constitution is the only D. 1, 2 and 3
democratic Constitution in world which
Answer: C
contains a list of citizens' duties.
Statement 2 is incorrect: Article 25 guarantees
On the contrary, the socialist countries gave rights of individuals, while Article 26 guarantees
equal importance to the fundamental rights rights of religious denominations or their sections.
and duties of their citizens. So, the In other words, Article 26 protects collective
Constitution of erstwhile USSR declared that freedom of religion. Like the rights under Article
the citizen's exercise of their rights and 25, the rights under Article 26 are also subject to
freedoms was inseparable from the public order, morality and health but not subject to
performance of their duties and obligations. other provisions relating to the Fundamental
So, Statement (2) is correct. Rights.
The 42nd Amendment Act of 1976 added 10 Additional Info:
Fundamental Duties to the Indian
Constitution. Article 25 says that all persons are equally entitled
to freedom of conscience and the right to freely
6th Amendment Act 2002 later added 11th profess, practice and propagate religion. It covers
Fundamental Duty to the list. Swaran Singh not only religious beliefs(doctrines) but also
Committee in 1976 recommended religious practices (rituals). Moreover, these rights
Fundamental Duties, the necessity of which are available to all persons—citizens as well as
was felt during the internal emergency of non-citizens. So, statement 1 is correct.
1975-77.
Article 27 lays down that no person shall be
compelled to pay any taxes for the promotion or
77. Which of the following statements is/are maintenance of any particular religion or religious
correct with reference to the right to freedom of denomination. In other words, the State should not
religion under the Indian Constitution? spend the public money collected by way of tax for
1. Article 25 covers both religious beliefs the promotion or maintenance of any particular
and religious practices. religion. This provision prohibits the State from
favouring, patronising and supporting one religion
2. Article 25 guarantees collective over the other. This means that the taxes can be
freedom of religion, while Article 26 used for the promotion or maintenance of all
protects rights of individuals. religions. So, statement 3 is correct.
3. Article 27 does not prohibit the state to
78. Consider the following statements about sitting of two Houses of Parliament
the Chairman of the Rajya Sabha: and the Chairman of Rajya Sabha
cannot preside over a joint sitting of
1. Presiding officer of the Rajya Sabha is
the two Houses of Parliament under
the ex-officio Chairman of the Rajya
any circumstances.
Sabha.
Statement 3 is correct: Unlike the Speaker
2. The Chairman of Rajya Sabha can
(who is a member of the House), the
preside over a joint sitting of the two
Chairman is not a member of the House. The
Houses of Parliament under special
Chairman cannot vote in the first instance.
circumstances.
However, he can cast a vote in the case of an
3. The Chairman cannot vote in the first equality of votes.
instance but can cast a vote in the case
Reference: M. Laxmikant.
of an equality of votes.
Which of the statements given above is/are
correct? 79. Consider the following statements about
the Leader of the Opposition:
A. 1 and 2 only
1. The office of the leader of the
B. 3 only
Opposition is mentioned in the
C. 1 and 3 only Constitution of India.
D. 1, 2 and 3 2. The leader of the largest Opposition
Answer: B party having not less than one-tenth
seats of the total strength of the House.
Explanation:
3. They are entitled to the salary,
Statement 1 is not correct: The presiding
allowances and other facilities
officer of the Rajya Sabha is known as the
equivalent to that of a cabinet minister.
Chairman. The vice-president of India is the
ex-officio Chairman of the Rajya Sabha. Which of the statements given above is/are
During any period when the Vice-President correct?
acts as President or discharges the functions A. 2 and 3 only
of the President, He does not perform the
B. 2 only
duties of the office of the Chairman of Rajya
Sabha. The Chairman of the Rajya Sabha can C. 1 and 2 only
be removed from his office only if he is D. 1, 2 and 3
removed from the office of the Vice-President.
Answer: A
Statement 2 is not correct:
Explanation:
• As a presiding officer, the powers, and
Statement 1 is not correct: The offices of the
functions of the Chairman in the Rajya
leader of the House and the leader of the
Sabha are similar to those of the
Opposition are not mentioned in the
Speaker in the Lok Sabha. However,
Constitution of India, they are mentioned in
the Speaker presides over a joint
the Rules of the House and Parliamentary Which of the statements given above is/are
Statute respectively. correct?
Leader of the House: A. 1 and 3 only
• Under the Rules of Lok Sabha, the B. 3 only
‘Leader of the House’ means the prime C. 1 and 2 only
minister, if he is a member of the Lok
Sabha or a minister who is a member D. 1, 2 and 3
of the Lok Sabha and is nominated by Answer: B
the prime minister to function as the Explanation:
Leader of the House.
Statement 1 is not correct: As per the
• There is also a ‘Leader of the House’ in provisions of the Constitution, the Speaker of
the Rajya Sabha. He is a minister and a the last Lok Sabha vacates his office
member of the Rajya Sabha and is immediately before the first meeting of the
nominated by the prime minister to newly-elected Lok Sabha. Therefore, the
function as such. President appoints a member of the Lok
Statement 2 is correct: Leader of the Sabha as the Speaker Pro Tem. Usually, the
Opposition: In each House of Parliament, senior most member is selected for this.
there is the ‘Leader of the Opposition’. The Statement 2 is not correct: The President
leader of the largest Opposition party having himself administers the oath to the Speaker
not less than one-tenth seats of the total Pro Tem. The Speaker Pro Tem has all the
strength of the House is recognized as the powers of the Speaker.
leader of the Opposition in that House.
• He also enables the House to elect the
Statement 3 is correct: His main functions are new Speaker. The date of the election
to provide constructive criticism of the of the Speaker is fixed by the President.
policies of the government and to provide an He also administers the floor test.
alternative government. They are also entitled
Statement 3 is correct: He presided over the
to the salary, allowances, and other facilities
first sitting of the newly-elected Lok Sabha.
equivalent to that of a cabinet minister.
His main duty is to administer the oath to the
Reference: M. Laxmikant. new members.
• When the new Speaker is elected by
80. Consider the following statements about the House, the office of the Speaker Pro
the Speaker Pro Tem: Tem ceases to exist. Hence, this office
is a temporary office, existing for a few
1. She/He is elected by the Lok Sabha
dates.
from amongst its members.
Reference: M. Laxmikant
2. Chief minister administers the oath to
the Speaker Pro Tem.
3. She/he presides over the first sitting 81. Consider the following statements:
of the newly-elected Lok Sabha.
1. The zero hour is mentioned in the may be addressed to a private member
Indian Constitution and Rules of if the subject matter of the question
Procedure. relates to some Bill, resolution, or other
matter connected with the business of
2. A starred question requires an oral
the House for which that member is
answer and hence supplementary
responsible. The procedure in regard
questions can follow.
to such questions is the same as that
3. An unstarred question, on the other followed in the case of questions
hand, requires a written answer. addressed to a minister.
Which of the statements given above is/are • The list of starred, unstarred, short
correct? notice questions and questions to
A. 1 and 3 only private members are printed in green,
B. 2, 3 only white, light pink, and yellow colour,
respectively, to distinguish them from
C. 1 and 2 only one another.
D. 1, 2 and 3 only Reference: M. Laxmikant.
Answer: B
Explanation: 82. Consider the following statements in the
Statement 1 is not correct: The zero hour is context of Calling attention motion:
not mentioned in the Indian Constitution and 1. It is introduced to deal with very
Rules of Procedure. Thus it is an informal important matters like the
device available to the members of the impeachment of the President.
Parliament to raise matters without any prior
2. A member can give more than two
notice.
calling attention notices for any one
Statement 2 is correct: The questions are of sitting.
three kinds, namely, starred, unstarred and
Which of the statements given above is/are
short notice. A starred question
correct?
(distinguished by an asterisk) requires an oral
answer and hence supplementary questions A. 1 only
can follow. B. 2 only
Statement 3 is correct: An unstarred question, C. Both 1 and 2
on the other hand, requires a written answer
D. Neither 1 nor 2
and hence, supplementary questions cannot
follow. A short notice question is one that is Answer: B
asked by giving a notice of fewer than ten Explanation:
days. It is answered orally.
Statement 1 is not correct:
• In addition to the ministers, the
Calling Attention Motion: It is introduced in
questions can also be asked to the
the Parliament by a member to call the
private members. Thus, a question
attention of a minister to a matter of urgent Statement 1 is correct: Council of Ministers
public importance and to seek an are appointed by the President on the advice
authoritative statement from him on that of Prime Minister
matter, for example, disturbances in any part Statement 2 is incorrect: Parliament decides
of the country, border troubles, railway the salary and allowances of the council of
accidents, shutting down of public ministers
undertakings.
Statement 3 is incorrect: Council of Ministers
Statement 2 is correct: It is an Indian along with the Prime Minister of India form
innovation in the parliamentary procedure 15% of the total strength of the lower house
and has been in existence since 1954. i.e. Lok Sabha. (The number cannot exceed
However, unlike the zero hour, it is 15%).
mentioned in the Rules of Procedure. A
member can not give more than two calling
attention notices for any one sitting. 84. Consider the following statements with
reference to Council of Ministers and the
Reference: M. Laxmikant.
President:
1. The advice tendered by the Council of
83. Consider the following statements with Ministers’ is binding on the President
reference to Council of Ministers:
2. This provision was introduced by the
1. Council of Ministers are appointed by 42nd Amendment Act 1976 and 44th
the President on the advice of Prime Amendment Act 1978.
Minister
3. The advice given by the council cannot
2. Prime Minister decides the salary and be inquired into by any court was
allowances of the council of ministers provided by 42nd and 44th
3. Council of Ministers along with the amendment acts.
Prime Minister of India form 15% of Which of the statements given above is/are
the total strength of the Parliament. correct?
A. 1 only
Which of the statements is/are correct? B. 2 and 3 only
A. 1 only C. 1 and 3 only
B. 2 and 3 only D. 1, 2 and 3
C. 2 only Answer: D
D. 1 and 3 only Statement 1 is correct: The advice tendered
Answer: A by the Council of Ministers’ is binding on the
President
Explanation:
Statement 2 is correct: This provision was Statement 3 is incorrect: Cabinet Committee
introduced by the 42nd Amendment Act 1976 is not a statutory body and is rather an extra-
and 44th Amendment Act 1978. constitutional body, which means they are
not mentioned in the constitution.
Statement 3 is incorrect: the advice is binding
on the President, and this provision was
introduced by the 42nd Amendment Act 1976 86. Council of Ministers includes which of
and 44th Amendment Act 1978. The acts also the following?
mentioned that the advice given by the
council cannot be inquired into by any court. 1. cabinet ministers
2. Ministers of state

85. Consider the following statements with 3. Deputy ministers


reference to difference between Cabinet 4. Cabinet Secretary
Committees and Council of Ministers
Select the correct code:
(COMs):
A. 1 and 2 only
1. Council of Ministers are vested with all
powers but only in theory. B. 3 and 4 only

2. Cabinet committees are collectively C. 1, 3 and 4 only


responsible to the Council of Ministers D. 1, 2 and 3 only
3. Cabinet Committees is a statutory Answer: D
body
Explanation:
Which of the above is/are correct?
The Council of Ministers is a wider body of 60
A. 1 only to 70 ministers including all 3 categories of
B. 2 and 3 only ministers, namely, cabinet ministers,
ministers of state, and deputy ministers.
C. 2 only
D. 1 and 3 only
87. Consider the following statements with
Answer: A
reference to cabinet secretary:
Explanation:
1. A cabinet secretary is appointed for a
Statement 1 is correct: Council of Ministers fixed tenure of two years.
(COMs) are vested with all powers but only
2. The cabinet secretariat is under the
in theory. It implements the decisions taken
direct charge of the prime minister.
by the cabinet while Cabinet Committees help
the cabinet in decision making. 3. Cabinet secretary is the ex-officio
chairman of the civil services board.
Statement 2 is incorrect: Council of Ministers
(COMs) are collectively responsible to the Which of the above is/are correct?
Lower House of the Parliament while there is A. 1 and 2 only
no such clause for cabinet committees.
B. 2 and 3 only
C. 1 and 3 only ensuring inter-ministerial
coordination, ironing out differences
D. 1, 2 and 3
amongst ministries or departments
Answer: D and evolving consensus through the
Explanation: instrumentality of the standing or ad
Statement 1 is correct: A cabinet secretary is hoc committees of secretaries.
appointed for a fixed tenure of two years. • Management of major crisis situations
Statement 2 is correct: The cabinet secretariat in the country and coordinating
is under the direct charge of the prime activities of various ministries in such
minister. a situation is also one of the functions
of the cabinet secretariat.
Statement 3 is correct: Cabinet secretary is the
ex-officio chairman of the civil services board. • Cabinet Secretariat is responsible for
the administration of the Government
Additional info: of India (Transaction of Business)
• A cabinet secretary is appointed for a Rules, 1961 and the Government of
fixed tenure of two years. India (Allocation of Business) Rules
1961, facilitating smooth transaction of
• According to All India Services (Death-
business in Ministries/ Departments
Cum-Retirement-Benefits) Rules, 1958,
of the Government.
the government can give extension in
service to a cabinet secretary provided
the total tenure does not exceed four
years.
88. The State Executive is made up of which
• As per the modified rules, the central of the following?
government may give an extension in
1. Governor
service for a further period not
exceeding three months, beyond the 2. Chief Minister
period of four years to a cabinet 3. Council of Ministers
secretary.
4. Advocate General of the State
Role of the cabinet secretary:
5. Non-Political Permanent Executive -
• The cabinet secretariat is under the Civil Servants
direct charge of the prime minister.
Select the correct code:
• The administrative head of the
A. 1 and 2 only
secretariat is the cabinet secretary who
is also the ex-officio chairman of the B. 3, 4 and 5 only
civil services board. C. 1, 3 and 4 only
Functions: D. 1, 2, 3, 4 and 5
• The cabinet secretariat assists in Answer: D
decision-making in government by
Explanation:
What is a State Executive? 90. Which of the following Statements is
incorrect with reference to the composition
• The government is based on the
of the Legislative Council?
separation of powers and consists of
three organs such as the Legislature, A. One-third of the Members of
Executive and the Judiciary. Legislative Councils are elected by the
state’s MLAs
• The state executive is envisaged with
the role of enforcing the laws made by B. Another 1/3rd by a special electorate
the legislature and looking after the comprising sitting members of local
governance in the state. governments such as municipalities
and district boards,
• The State Executive is made up of the
following: C. 1/12th by an electorate of teachers and
another 1/12th by registered
o Governor
graduates.
o Chief Minister
D. The remaining members are appointed
o Council of Ministers by the Chief Minister for distinguished
o Advocate General of the State services in various fields

o Non-Political Permanent Answer: D


Executive - Civil Servants Explanation:
Statement 1 is correct: One-third of the MLCs
89. Which of the following States does not are elected by the state’s MLAs
have a State Legislative Council? Statement 2 is correct: Another 1/3rd by a
A. Andhra Pradesh special electorate comprising sitting members
of local governments such as municipalities
B. Karnataka
and district boards
C. Tamil Nadu
Statement 3 is correct: 1/12th by an electorate
D. Telangana of teachers and another 1/12th by registered
Answer: C graduates.
Explanation: Statement 4 is incorrect: The remaining
members are appointed by the Governor for
• As of 2022, 6 out of 28 states have a
distinguished services in various fields
State Legislative Council.
namely, literature, science, art, cooperative
• These are Andhra Pradesh, movement, and social service.
Karnataka, Telangana, Maharashtra,
Additional info:
Bihar, and Uttar Pradesh.
• India has a bicameral parliamentary
system, which means it has two houses
of parliament: the Lok Sabha (House of
the People) and the Rajya Sabha
(Council of States). The Legislative A. 1 only
Council, on the other hand, is the B. 2 only
upper house of the state legislature in
certain states of India. C. Both 1 and 2

• Currently, six states in India have D. Neither 1 nor 2


Legislative Councils, not seven. These Answer: C
states are Andhra Pradesh, Bihar,
Explanation:
Jammu and Kashmir, Karnataka,
Maharashtra, and Uttar Pradesh. Statement 1 is correct: A money bill can only
Telangana Legislative Council was be introduced in the Vidhan Sabha.
abolished in 2019. Thank you for Statement 2 is correct: Money bills cannot be
bringing this to my attention. introduced by a private member.
• The members of the Legislative Additional info:
Council are not directly elected by the
Money bill is passed in State Legislature in the
people. Instead, they are elected by
following ways:
different methods, including indirect
elections by members of local bodies, • A money bill can only be introduced in
teachers, graduates, and others, as well the Vidhan Sabha. The Money bill is
as by appointment by the Governor of inclusive of expenditure that is
the state. authorized by the government,
imposition or abolition of taxes,
• The powers and functions of the
borrowing, etc.
Legislative Council are similar to those
of the Rajya Sabha at the national level. • Money bills cannot be introduced by a
They include scrutinizing and private member. It has to be
approving legislation, debating issues introduced by a Minister on the
of public importance, and examining recommendations of the Governor. In
the work of the government. However, case of any confusion with regards to
in the event of a disagreement between the certification of that particular bill
the two houses of the state legislature, being a money bill, the Speaker of the
the Legislative Assembly, being the Vidhan Sabha would determine the
lower house, has the final say. same.
• This money bill after being passed by
the Vidhan Sabha needs to be sent to
91. Consider the following statements with
the Vidhan Parishad. This bill needs to
reference to Money Bill In State Legislature:
be returned within the time span of 14
1. A money bill can only be introduced in days, with, or without any
the Vidhan Sabha. recommendations. The Vidhan Sabha
2. Money bills cannot be introduced by a may accept or reject these
private member. recommendations. After these stages

Which of the above is/are correct?


are completed, this bill is sent to the between this two w.r.t. Ordinance making is
Governor for his assent. given below:

Ordinance
92. Consider the following statements with Ordinance Making
Making Power of
reference to Ordinance Making Power of Power of Governor
President
Governor:
1. Governor can roll-out an ordinance
He can promulgate
for only those matters on which state
ordinance when the
legislature can make laws
He can Legislative Assembly
2. His power to promulgate ordinance is promulgate is not in session in
a discretionary power. ordinance when case of the
3. His ordinances have the same effect on either Lok Sabha unicameral
policies as state’s acts will have or Rajya Sabha is legislature or when
not in session or both Legislative
Which of the above is/are correct?
if both are not in assembly and council
A. 1 only session are not in session in
B. 2 and 3 only case of a bicameral
legislature
C. 2 only
D. 1 and 3 only
He can roll-out an
Answer: D ordinance for He can roll-out an
Explanation: only those ordinance for only
matters on which those matters on
Statement 1 is correct: Governor can roll-out
Parliament (Lok which state
an ordinance for only those matters on which
Sabha & Rajya legislature can make
state legislature can make laws
Sabha) can make laws
Statement 2 is incorrect: His power to laws
promulgate ordinance is not a discretionary
power. Council of Ministers’ (headed by CM)
His ordinances have
advice is a prerequisite
the same effect on
Statement 3 is correct: His ordinances have policies as state’s acts
the same effect on policies as state’s acts will His ordinances
will have. If his
have. have the same
ordinance legislates
effect on policies
Additional info: on matters which
as parliament’s
state government has
Article 213 deals with the power of the acts will have
no power on, the
Governor to legislate through ordinances. His
ordinance stands null
power of ordinance making is quite similar to
and void
the President’s power. The comparisons
The ordinance
The ordinance 93. Consider the following statements regarding
introduced by
introduced by him fifth schedule of the Indian Constitution:
him can be
can be withdrawn
withdrawn 1. It is valid for every state
anytime
anytime 2. It deals with tribals areas and scheduled
tribes
His power to Which of the statements is/are correct?
His power to
promulgate
promulgate A. 1 only
ordinance is not a
ordinance is not a B. 2 only
discretionary
discretionary power.
power. Council of C. both the statements
Council of Ministers’
Ministers’
(headed by CM) D. none of the above
(headed by PM)
advice is a pre- Answer: D
advice is a pre-
requisite
requisite Explanation:
• Article 244 in Part X of the Constitution
President’s envisages a special system of
instructions on the administration for certain areas designated
following three cases as ‘scheduled areas’ and ‘tribal areas’.
are must:
• The Fifth Schedule of the Constitution
If a bill containing the deals with the administration and control
same provisions of scheduled areas and scheduled tribes in
would have required
any state except the four states of Assam,
the previous sanction
of the President for its Meghalaya, Tripura and Mizoram. Thus
introduction into the both the statement are incorrect
No instructions
state legislature Additional info:
are needed by
If he would have
President’s when • The Sixth Schedule of the Constitution, on
deemed it necessary
he promulgates the other hand, deals with the
to reserve a bill
an ordinance containing the same administration of the tribal areas in the four
provisions for the northeastern states of Assam, Meghalaya,
consideration of the Tripura and Mizoram
President
Fifth Schedule
If an act of the state
legislature containing • Declaration of Scheduled Areas: The
the same provisions president is empowered to declare an area
would have been to be a scheduled area. He can also increase
invalid without or decrease its area, alter its boundary lines,
receiving the rescind such designation, or make fresh
President’s assent
orders for such designation on an area in
consultation with the governor of the state B. Ministry of Home Affairs
concerned. C. Parliament
• Executive Power of State and Centre: The D. Governor of the State
executive power of a state extends to the
scheduled areas therein. But the governor Answer: D
has a special responsibility regarding such Explanation:
areas. He has to submit a report to the Law applicable to Scheduled Areas under Fifth
president regarding the administration of Schedule : The governor is empowered to direct
such areas, annually or whenever so that any particular act of Parliament or the state
required by the President. legislature does not apply to a scheduled area or
• Tribes Advisory Council: Each state having apply with specified modifications. Thus option D
scheduled areas has to establish a tribes is correct
advisory council to advise on welfare and Additional info:
advancement of the scheduled tribes.
• Article 244 in Part X of the Constitution
• A similar council can also be established in envisages a special system of
a state having scheduled tribes but not administration for certain areas designated
scheduled areas therein, if the president so as ‘scheduled areas’ and ‘tribal areas’.
directs.
• The Fifth Schedule of the Constitution
• Law applicable to Scheduled Areas: The deals with the administration and control
governor is empowered to direct that any of scheduled areas and scheduled tribes in
particular act of Parliament or the state any state except the four states of Assam,
legislature does not apply to a scheduled Meghalaya, Tripura and Mizoram. Thus
area or apply with specified modifications both the statement are incorrect
and exceptions. He can also make
• Fifth Schedule
regulations for the peace and good
government of a scheduled area after o Declaration of Scheduled Areas:
consulting the tribes advisory council. The president is empowered to
declare an area to be a scheduled
• The Constitution requires the president to
area. He can also increase or
appoint a commission to report on the
decrease its area, alter its boundary
administration of the scheduled areas and
lines, rescind such designation or
the welfare of the scheduled tribes in the
make fresh orders for such
states
designation on an area in
Source – Notes, Laxmikanth. consultation with the governor of
the state concerned.

94. Who/Which of the following is empowered to o Executive Power of State and


direct that a law of Parliament does not apply to a Centre: The executive power of a
scheduled area? state extends to the scheduled areas
A. President of India therein. But the governor has a
special responsibility regarding B. 2 only
such areas. He has to submit a C. both the statements
report to the president regarding
the administration of such areas, D. none of the above
annually or whenever so required Answer: D
by the President.
Statement 1 is incorrect as the Government of
o Tribes Advisory Council: Each state India Act of 1935 set up the Federal Court of India
having scheduled areas has to to act as an intermediate appellate between High
establish a tribes advisory council courts and the Privy Council. The Federal Court
to advise on welfare and continued to function till 26th January 1950, when
advancement of the scheduled independent India’s Constitution came into force
tribes.
Statement 2 is incorrect as the Supreme Court of
▪ A similar council can also be India was inaugurated on January 28, 1950 and
established in a state having replaced the Federal Court. It was not renamed as
scheduled tribes but not his jurisdiction was greater as it also replaced the
scheduled areas therein, if
Privy Council. It is the apex court and the highest
the president so directs.
court of appeal in the country.
o and exceptions. He can also make
Additional info:
regulations for the peace and good
government of a scheduled area Articles 124 to 147 in Part V of the Constitution
after consulting the tribes advisory deals with its organisation, independence,
council. jurisdiction, powers, procedures

o The Constitution requires the Source – Notes.


president to appoint a commission
to report on the administration of 96. Consider the following statements:
the scheduled areas and the welfare
of the scheduled tribes in the states 1. A retired Supreme Court Judge can act as a
Judge of the Supreme Court
Source – Notes, Laxmikanth.
2. President can appoint a judge of a High
Court as an ad hoc judge of SC for a
95. Consider the following statements temporary period
1. Federal Court of India established under Which of the statements given above is / are
Government of India Act, 1935 functioned correct?
till 15th August, 1947 A. Only 1
2. The Federal court was renamed as Supreme B. Only 2
Court of India on 26th January, 1950 after
enforcement of Indian Constitution C. Both 1 and 2

Which of the statements is/are correct? D. Neither 1 nor 2

A. 1 only Answer: A
Explanation: C. 1 and 3 only
Statement 1 is correct: Retired Judge - At any time, D. All of the above
the CJI can request a retired judge of the SC or a
Answer: C
retired judge of a HC (who is duly qualified for
appointment as a judge of SC) to act as a judge of Explanation:
SC for a temporary period. He can do so only with • Statement 3 is correct: A judge of the
the previous consent of the President and also of Supreme Court can be removed from his
the person to be so appointed. Such a judge is Office by an order of the president.
entitled to such allowances as the President may
• Statement 2 is incorrect: The President can
determine issue the removal order only after an
Statement 2 is incorrect: Ad hoc Judge - When address by Parliament has been presented
there is a lack of quorum of the permanent judges, to him in the same session for such
the CJI can appoint a judge of a High Court as an removal.
ad hoc judge of SC for a temporary period. He can • The address must be supported by a special
do so only after consultation with the chief justice majority of each House of Parliament (ie, a
of the High Court concerned and with the previous majority of the total membership of that
consent of the President. The judge so appointed House and a majority of not less than two-
should be qualified for appointment as a judge of thirds of the members of that House
the Supreme Court. present and voting). The grounds of
Additional info: Acting CJI - A judge of the SC can • removal are two–proved misbehaviour or
be appointed as an acting CJI by the President in incapacity
case of temporary vacancy, absence or inability of
• Statement 1 is correct: The Judges Enquiry
CJI to perform his duties.
Act (1968) regulates the procedure relating
Source – Notes, Laxmikanth. to the removal of a judge of the Supreme
Court by the process of impeachment:

97. Consider the following statements regarding o A removal motion signed by 100
removal of judge: members (in the case of Lok Sabha)
or 50 members (in the case of Rajya
1. Process of impeachment has been framed
Sabha) is to be given to the
under Judges Enquiry Act, 1968
Speaker/Chairman.
2. Judge can be removed only if collegium of
o The Speaker/Chairman may admit
supreme court recommend government to
the motion or refuse to admit it.
start the process
o If it is admitted, then the
3. Judge of Supreme court is removed by
Speaker/Chairman is to constitute
President
a three-member committee to
Which of the statements is/are correct? investigate into the charges.The
A. 1 and 2 only committee should consist of
B. 2 and 3 only
▪ (a) the chief justice or a high court and a hierarchy of subordinate
judge of the Supreme Court, courts. The high court occupies the top
position in the judicial administration of a
▪ (b) a chief justice of a high
state.
court, and
Strength of HC –
▪ (c) a distinguished jurist
• Statement 1 is incorrect: Every high
o If the committee finds the judge to
court (whether exclusive or common)
be guilty of misbehaviour or
consists of a chief justice and such
suffering from an incapacity, the
other judges as the President may
House can take up the
from time to time deem necessary to
consideration of the motion.
appoint. (In case of SC, Parliament
o After the motion is passed by each decides). Also note their is no such
House of Parliament by special ministry with name Ministry of Legal
majority, an address is presented to Affairs, it's ministry of law and justice
the President for removal of the
• Thus, the Constitution does not
judge.
specify the strength of a high court
o Finally, the President passes an
• Statement 2 is correct: Under Original
order removing the judge.
Jurisdiction, HC has power to punish for
Source – Notes, Laxmikanth. contempt of court. Under Contempt of
Courts Act, 1971, a High Court shall have
jurisdiction to inquire into or try a
98. Consider the following statements:
contempt of itself or of any court
1. The strength of High Court are specified by subordinate to it, whether the contempt is
the Ministry of Legal Affairs alleged to have been committed within or
2. High court also has a power to punish over outside the local limits of its jurisdiction,
contempt of court and whether the person alleged to be guilty
of contempt is within or outside such
Which of the statements given above is / are
limits.
correct?
Additional info:
A. Only 1
Other power of HC
B. Only 2
• Matters of admiralty, will, marriage,
C. Both 1 and 2
divorce, company laws and contempt
D. Neither 1 nor 2 of court.
Answer: B • Disputes relating to the election of
• In the Indian single integrated judicial members of Parliament and state
system, the high court operates below the legislatures.
Supreme Court but above the subordinate • Regarding revenue matters or an act
courts. The judiciary in a state consists of a ordered or done in revenue collection.
• Enforcement of fundamental rights of • This power of superintendence of a
citizens. high court is very broad because –
• Cases ordered to be transferred from a • It extends to all courts and tribunals
subordinate court involving the whether they are subject to appellate
interpretation of the Constitution to • jurisdiction of HC or not
its own file.
• It covers not only administrative
• The four high courts (i.e., Calcutta, superintendence but also judicial
Bombay, Madras and Delhi High superintendence;
Courts) have original civil jurisdiction
in cases of higher value. • It is a revisional jurisdiction; and

Writ Jurisdiction • It can be suo-motu (on its own) and


not necessarily on the application of a
• Article 226 empowers HC to issue
party
writs for the enforcement of FRs of
citizens and for any other purpose.
• Writ jurisdiction of HC is co-terminus 99. The writ jurisdiction of High Courts is wider
with SC and in fact extends beyond than Supreme court as per constitution as it
expands to which matters:
fundamental rights to include
ordinary legal right as well. A. Civil and criminal matters
Appellate Jurisdiction B. Fundamental and other legal rights
• A high court is primarily a court of C. Fundamental rights and matters in appeals
appeal. It hears appeals against the D. Fundamental rights and stay matters
judgments of subordinate courts
functioning in its territorial Answer: B
jurisdiction. It has appellate Explanation:
jurisdiction in both civil and criminal Option B is correct: However, the writ jurisdiction
matters. Hence, the appellate of the high court is wider than that of the Supreme
jurisdiction of a high court is wider Court. This is because the Supreme Court can issue
than its original jurisdiction. writs only for the enforcement of fundamental
• Appeals from the decisions of the rights and not for any other purpose, that is, it does
administrative and other tribunals lie not extend to a case where the breach of an
to the division bench of the state high ordinary legal right is alleged.
court. Additional Info:
Supervisory Jurisdiction • Article 226 of the Constitution empowers a
• HC has power of superintendence high court to issue writs including habeas
over all courts and tribunals corpus, mandamus, certiorari, prohibition
functioning in its territorial and quo warranto for the enforcement of
jurisdiction (except military courts or the fundamental rights of the citizens and
tribunals) for any other purpose.
• The phrase ‘for any other purpose’ refers to • issued by a higher court to a lower
the enforcement of an ordinary legal right. court
Thus writ jurisdiction extends to both • prevent the latter from exceeding
fundamental and legal rights of citizens jurisdiction or usurping a
• The high court can issue writs to any jurisdiction that it does not
person, authority and government not only possess
within its territorial jurisdiction but also • unlike mandamus that directs
outside its territorial jurisdiction if the activity it prohibits and directs
cause of action arises within its territorial inactivity
jurisdiction
• Certiorari – means “to be certified”
• The writ jurisdiction of the high court
(under Article 226) is not exclusive but • issued by higher court to a lower
concurrent with the writ jurisdiction of the court to transfer a case pending or
Supreme Court (under Article 32). It squash order
means, when the fundamental rights of a • grounds: excess of jurisdiction or
citizen are violated, the aggrieved party has lack or error of law
the option of moving either the high court
• Quo Warranto – means “by what
or the Supreme Court directly.
authority or warrant”
Various Writs
• issued by the court to enquire into
• Habeas Corpus legality of claim of a person to a
• Latin term meaning “to have the public office
body of” • prevent illegal assassination of
• issued against both public and public office by person
private • unlike other four can be sought by
• order issued by a court to a person any interested person and not only
who has detained another to aggrieved
produce the body of the latter Source - Laxmikanth
before it.
• Mandamus – means “we command”
100. Consider the following statements regarding
• issued by the court to make a Sedition Law:
public official perform his official 1. Proceedings under sedition law has been
duties that he has failed or refuse suspended by Supreme Court as of now
to perform
2. Sedition is a non-bailable offense
• issued against any public body,
Corp, inferior court, tribunal, or 3. Section 124A was not part of original law of
government. IPC enacted in 1860
Which of the statements is/are correct?
• Prohibition – means “to forbid”
A. 1 and 2 only philosophy which motivates judges
to depart from the traditional
B. 2 and 3 only
precedents in favour of progressive
C. 1 and 3 only and new social policies”.
D. All of the above • About Sedition Law
Answer: D o Section 124A of the Indian Penal
Explanation: Code lays down the punishment for
sedition. The IPC was enacted in
• Statement 1 is correct: The Supreme Court
1860, under the British Raj.
suspended pending criminal trials and
court proceedings under Section 124A o Throughout the Raj, this section
(sedition) of the Indian Penal Code, while was used to suppress activists in
allowing the Union of India to reconsider favor of national independence,
the British-era law. including Tilak and Mahatma
Gandhi, both of whom were found
• Statement 3 is correct: Although Thomas
guilty and imprisoned.
Macaulay, who drafted the Indian Penal
Code, had included the law on sedition, it o Section 124A defines sedition as:
was not added in the code enacted in 1860. ▪ An offence committed
Legal experts believe this omission was when “any person by
accidental. In 1890, sedition was included words, either spoken or
as an offence under section 124A IPC written, or by signs, or by
through the Special Act XVII visible representation, or
• All pending trials, appeals and proceedings otherwise, brings or
with respect to the charge framed under attempts to bring into
Section 124A of the IPC be kept in hatred or contempt, or
temporary suspension. excites or attempts to excite
disaffection towards the
o The court also restrained centre and
government established by
states from registering FIRs,
law in India”.
continuing investigations or taking
coercive measures under Section ▪ Disaffection includes
124A. disloyalty and all feelings of
enmity.
• This judgement is seen as judicial activism
by many experts as the article indirectly o However, comments without
violated article 19 and 21 of the exciting or attempting to excite
Constitution. hatred, contempt or disaffection,
will not constitute an offense.
• About Judicial Activism
o Judicial activism is a dynamic o Statement 2 is correct: Sedition is a
process of judicial outlook in a non-bailable offense.
changing society. It is a " judicial
o Punishment under Section 124A
ranges from imprisonment up to
three years to a life term
with/without a fine.

You might also like